You are on page 1of 98

DIRECTIONS: Each item below contains a question or incomplete statement followed by suggested

responses. Select the one best response to each question.

1. A 42-year-old man comes to the emergency room with the chief complaint that “the men are
following me.” He also complains of hearing a voice telling him to hurt others. He tells the
examiner that the news anchorman gives him special messages about the state of the world
every night through the TV. This last belief is an example of which of the following
psychiatric findings?

a. Grandiose delusion
b. Illusion
c. Loose association
d. Idea of reference
e. Clouding of consciousness

2. A 32-year-old woman is seen in an outpatient psychiatric clinic for the chief complaint of a
depressed mood for four months. During the interview, she gives very long, complicated
explanations and many unnecessary details before finally answering the original questions.
This style of train of thought is an example of which of the following psychiatric findings?

a. Loose association
b. Circumstantiality
c. Neologism
d. Perseveration
e. Flight of ideas

3. A 23-year-old man is brought to the emergency room after he walked up to a stranger in a


grocery store and began hitting her, claiming that he “knew that she had been sent to spy on
him.” In the emergency room he was found to be alert and oriented to person, place, and
time. He denied hearing voices. He continued to insist that there were “special agents”
planted throughout the city to watch him. This symptom is an example of which of the
following psychiatric findings?

a. Delusion
b. Illusion
c. Perceptual representation of a sound or an image not actually present
d. Egomania
e. Dissociative fugue

4. A 55-year-old man is brought to the psychiatrist by his wife after she found him wandering
outside their home wearing only his underwear. On exam, the patient notes that his memory
is “not as good as it used to be.” Which of the following tests is most likely to be helpful in the
diagnosis of this patient?
a. EEG
b. MRI
c. Serum glucose
d. Serum amylase
e. Urinary myoglobin

5. A 56-year-old man has been hospitalized for a myocardial infarction. Two days after
admission, he awakens in the middle of the night and screams that there is a man standing by
the window in his room. When the nurse enters the room and turns on a light, the patient is
relieved to learn that the “man” was actually a drape by the window. This misperception of
reality is best described by which of the following psychiatric terms?

a. Delusion
b. Hallucination
c. Illusion
d. Projection
e. Synesthesia

6. A 72-year-old woman is admitted to the burn unit with second- and third-degree burns
covering 35% of her body, which she received in a house fire. At 8 P.M. on the fourth day of
her hospital stay, she pulls out her IV and begins screaming that people are trying to hurt her.
Several hours later she is found to be difficult to arouse and disorented. Which of the
following is the most likely diagnosis?

a. Emergence of an underlying dementia


b. Brief reactive psychosis
c. Acute manic episode
d. Delirium
e. Acute stress disorder

7. A psychiatric resident is called to consult on the case of a 75-year-old woman who had
undergone a hip replacement two days before. On examination, the resident notes that the
patient states the date as 1956, and she thinks she is at her son’s house. These impairments
illustrate which aspect of the mental status examination?

a. Concentration
b. Memory
c. Thought process
d. Orientation
e. Level of consciousness

8. A 52-year-old man is sent to see a psychiatrist after he is disciplined at his job because he
consistently turns in his assignments late. He insists that he is not about to turn in anything
until it is “perfect, unlike all of my colleagues.”He has few friends because he annoys them
with his demands for “precise timeliness” and because of his lack of emotional warmth. This
has been a lifelong pattern for the patient, though he refuses to believe the problems have
anything to do with his personal behavior. Which of the following is the most likely
diagnosis for this patient?

a. Obsessive-compulsive disorder
b. Obsessive-compulsive personality disorder
c. Borderline personality disorder
d. Bipolar disorder, mixed state
e. Anxiety disorder not otherwise specified

9. A 23-year-old woman comes to the psychiatrist because she “cannot get out of the shower.”
She tells the psychiatrist that she has been unable to go to her job as a secretary for the past
three weeks because it takes her at least four hours to shower. She describes an elaborate
ritual in which she must make sure that each part of her body has been scrubbed three times,
in exactly the same order each time. She notes that her hands are raw and bloody from all the
scrubbing. She states that she hates what she is doing to herself but becomes unbearably
anxious each time she tries to stop. She notes that she has always taken long showers, but the
problem has been worsening steadily for the past five months. She mentions that she also has
a checking ritual when locking her doors (each time she locks the door she has to check five
times that it is indeed locked). She denies problems with friends or at work, other than the
problems that currently are keeping her from going to work. Which of the following is the
most likely diagnosis?

a. Attention deficit hyperactivity disorder


b. Obsessive-compulsive disorder
c. Obsessive-compulsive personality disorder
d. Separation anxiety disorder
e. Brief psychotic disorder

10. A 21-year-old woman comes to the psychiatrist because she is afraid she is “losing her mind.”
The patient notes that the symptoms first began two months previously while she was riding
home on the subway. She now avoids riding the subway because she is afraid the symptoms
will recur there. She experiences “waves of anxiety” that occur randomly approximately four
to five times per day. She denies the use of drugs or alcohol, and her physical examination is
normal. Which of the following is the most likely diagnosis?
a. Panic disorder
b. Generalized anxiety disorder
c. Schizoid personality disorder
d. Schizotypal personality disorder
e. Anxiety secondary to a general medical condition
f. Factitious disorder
g. Malingering
h. Schizophreniform disorder
i. Schizophrenia
11. A 28-year-old man comes to the psychiatrist because his employer required it. The patient
says that he does not know why the employer required it—that his job is good and that he
likes it because it requires him to sit in front of a computer screen all day. He notes he has
one friend whom he has had for over 20 years and “doesn’t need anyone else.” The friend
lives in another state and the patient has not seen him for at least a year. The patient denies
any psychotic symptoms. His eye contact is poor and his affect is almost flat. Which of the
following is the most likely diagnosis?
a. Panic disorder
b. Generalized anxiety disorder
c. Schizoid personality disorder
d. Schizotypal personality disorder
e. Anxiety secondary to a general medical condition
f. Factitious disorder
g. Malingering
h. Schizophreniform disorder
i. Schizophrenia

12. An 18-year-old man is brought to the emergency room by his college roommate, after the
roommate discovered that the patient had not left his room for the past three days, neither to
eat nor to go to the bathroom. The roommate noted that the patient was kind of “weird.”
Mental status examination reveals that the patient has auditory hallucinations of two voices
commenting upon his behavior. The patient’s parents note that their son has always been
somewhat of a loner and unpopular, but otherwise did fairly well in school. Which of the
following is the most likely diagnosis?

a. Panic disorder
b. Generalized anxiety disorder
c. Schizoid personality disorder
d. Schizotypal personality disorder
e. Anxiety secondary to a general medical condition
f. Factitious disorder
g. Malingering
h. Schizophreniform disorder
i. Schizophrenia

13. A 32-year-old woman comes to the psychiatrist with a chief complaint of anxiety. She notes
that she worries about paying the mortgage on time, whether or not she will get stuck in
traffic and be late for appointments, her husband’s and daughter’s health, and the war in Iraq.
She notes that she has always been anxious, but since the birth of her daughter two years ago,
the anxiety has worsened to the point that she feels she cannot function as well as she did
previously. Which of the following is the most likely diagnosis?

a. Panic disorder
b. Generalized anxiety disorder
c. Schizoid personality disorder
d. Schizotypal personality disorder
e. Anxiety secondary to a general medical condition
f. Factitious disorder
g. Malingering
h. Schizophreniform disorder
i. Schizophrenia

14. A 23-year-old woman comes to the emergency room with the chief complaint that she has
been hearing voices for seven months. Besides the hallucinations, she has the idea that the
radio is giving her special messages. When asked the meaning of the proverb “People in glass
houses should not throw stones,” the patient replies, “Because the windows would break.”
Which of the following mental status findings does this patient display?

a. Poverty of content
b. Concrete thinking
c. Flight of ideas
d. Loose associations
e. Autistic thinking

15. A 69-year-old man is brought to see his physician by his wife. She notes that over the past
year he has experienced a slow, stepwise decline in his cognitive functioning. One year ago
she felt his thinking was “as good as it always had been,” but now he gets lost around the
house and can’t remember simple directions. The patient insists that he feels fine, though he
is depressed about his loss of memory. He is eating and sleeping well. Which of the following
is the most likely diagnosis?

a. Multi-infarct dementia
b. Mood disorder secondary to a general medical condition
c. Schizoaffective disorder
d. Delirium
e. Major depression

16. A 50-year-old woman becomes depressed after her son dies in a carcrash. She and her
husband are devastated by the loss. Although the patient has been in otherwise good health,
she has lost 10 lb in the last two months. She tells the physician that she would be better off
dead. Which of the following statements with regard to the assessment of suicide risk in this
patient is true?

a. The patient’s gender puts her at higher risk for a completed suicide than if she were male
b. The patient is unlikely to commit suicide because she was able to talk about it
c. The patient’s age puts her at higher risk than a younger (<40 years old) patient
d. The patient’s good health puts her at higher risk for a successful attempt at suicide
e. The patient’s marital status puts her at higher risk of suicide than if she were single
17. A 24-year-old man is admitted to the inpatient psychiatry unit after his mother observed him
standing in place for hours at a time in abnormal postures. During his exam, the patient
stands with one arm raised directly above his head and the other straight out in front of him.
He is mute, does not appear aware of his surroundings, and actively resists any attempts to
change his position. Which of the following best describes the patient’s behavior?

a. Apraxia
b. Dystonia
c. Synesthesia
d. Catatonia
e. Trance state

18. A 36-year-old woman with schizophrenia comes to the emergency room with the chief
complaint that “they are trying to kill me.” In the examining room, she is hypervigilant and
insists on sitting in the corner with her back to the wall. Suddenly she begins to stare intently
into the corner and say, “No, you can’t make me do that!” Which of the following symptoms
is this patient most likely experiencing?

a. Concrete thinking
b. Depersonalization
c. Flight of ideas
d. Hallucination
e. Idea of reference

19. A patient is able to appreciate subtle nuances in thinking and can use metaphors and
understand them. This patient’s thinking can be best defined by which of the following
terms?

a. Intellectualization
b. Abstract
c. Rationalization
d. Concrete
e. Isolation of affect

20. A 65-year-old man, who had been hospitalized for an acute pneumonia three days previously,
begins screaming for his nurse, stating that “there are people in the room out to get me.” He
then gets out of bed and begins pulling out his IV line. On exam, he alternates between
agitation and somnolence. He is not oriented to time or place. His vital signs are as follows:
pulse, 126 beats per minute; respiration, 32 per minute; blood pressure (BP), 80/58;
temperature, 39.2°C (102.5°F). Which of the following diagnoses best fits this patient’s clinical
picture?

a. Dementia
b. Schizophreniform disorder
c. Fugue state
d. Delirium
e. Brief psychotic episode

21. A 35-year-old man is brought to see a psychiatrist by his wife, who states that her husband
keeps getting lost, even in places he has been familiar with for years. The patient’s father was
institutionalized and died at age 37. On exam, the patient is oriented to person only. He
cannot accurately make change for a dollar, though he used to work as a banker. Which of
the following diagnostic tests would be most useful for this patient?

a. EEG
b. Liver function tests
c. Serum amylase
d. Blood toxicology screen
e. Magnetic resonance imaging (MRI)

22. A 34-year-old woman comes to her physician with the chief complaint of a depressed mood.
She also notes trouble concentrating, hypersomnia, a 20-lb weight gain, and slowed
mentation. Which of the following diagnostic tests will be most helpful in diagnosing this
patient?

a. Thyroid function tests


b. Liver function tests
c. Serum ceruloplasmin
d. EEG
e. Urine amino acids

23. A 23-year-old man presents to the emergency room with the history of a fever up to 38°C
(100.5°F) intermittently over the past two weeks, a persistent cough, and a 10-lb weight loss
in the past month. He notes that he has also been becoming increasingly forgetful for the past
month and that his thinking is “not always clear.” He has gotten lost twice recently while
driving. Which of the following diagnostic tests will be most helpful with this patient?

a. EEG
b. Liver function tests
c. Thyroid function tests
d. HIV antibody test
e. Skull x-ray

24. A 27-year-old woman comes to her physician because she has passed out several times in the
past three weeks. The patient states that she does not remember what happens, but suddenly
she “wakes up on the floor.” Her husband notes that each of these episodes has occurred in
the middle of a fight between the two. He says that his wife will suddenly crumple to the
floor, “jerk all over,” and remain that way for approximately five minutes. When she wakes
up, he says, she is fully alert and oriented. Which of the following diagnostic tests will be
most helpful with this patient?
a. EEG
b. ECG
c. MRI
d. Dexamethasone suppression test
e. Serum amylase

25. A 19-year-old woman presents to the emergency room with the chief complaint of a
depressed mood for two weeks. She notes that since her therapist went on vacation she has
experienced suicidal ideation, crying spells, and an increased appetite. She states that she has
left 40 messages on the therapist’s answering machine telling him that she is going to kill
herself and that it would serve him right for leaving her. Physical exam reveals multiple well-
healed scars and cigarette burns on the anterior aspect of both forearms. Which of the
following diagnoses best fits this patient’s clinical presentation?

a. Dysthymic disorder
b. Bipolar disorder
c. Panic disorder
d. Borderline personality disorder
e. Schizoaffective disorder

26. A 28-year-old man comes to the physician with the chief complaint that he has been
depressed for years. He notes that his mood is never good, though he has never seriously
considered suicide. He often feels hopeless and has problems concentrating. His sleep pattern
and appetite have not changed. He denies hallucinations of any kind. Based on this clinical
picture, which of the following is the most likely diagnosis?

a. Conversion disorder
b. Avoidant personality disorder
c. Dysthymic disorder
d. Major depression
e. Adjustment disorder

27. A 29-year-old man is brought to the emergency room by his wife after he woke up with
paralysis of his right arm. The patient reports that the day before, he had gotten into a verbal
altercation with his mother over her intrusiveness in his life. The patient notes that he has
always had mixed feelings about his mother, but that people should always respect their
mothers above all else. Which of the following diagnoses best fits this patient’s clinical
picture?

a. Major depression
b. Conversion disorder
c. Histrionic personality disorder
d. Fugue state
e. Adjustment disorder
28. A 28-year-old business executive sees her physician because she is having difficulty in her
new position, as it requires her to do frequent public speaking. She states that she is terrified
she will do or say something that will cause her extreme embarrassment. The patient says that
when she must speak in public, she becomes extremely anxious and her heart beats
uncontrollably. Based on this clinical picture, which of the following is the most likely
diagnosis?

a. Panic disorder
b. Avoidant personality disorder
c. Specific phobia
d. Agoraphobia
e. Social phobia

29. A 24-year-old man comes to the physician with the chief complaint that his nose is too big, to
the point of being hideous. The patient states that his nose is a constant embarrassment to
him, and he would like it surgically reduced. He tells the physician that three previous
surgeons had refused to operate on him because they said his nose was fine, but the patient
asserts that “they just didn’t want such a difficult case.” The physician observes that the
patient’s nose is of normal size and shape. Based on this patient’s clinical picture, which of the
following is the most likely diagnosis?

a. Schizophrenia
b. Narcissistic personality disorder
c. Body dysmorphic disorder
d. Anxiety disorder not otherwise specified
e. Schizoaffective disorder

30. Which of the following terms best fits the definition “the proportion of a population affected
by a disorder at a given time?

a. Prevalence
b. Incidence
c. Validity
d. Reliability
e. Relative risk

31. A 56-year-old man is brought to the physician’s office by his wife because she has noted a
personality change during the past three months. While the patient is being interviewed, he
answers every question with the same three words. Which of the following symptoms best
fits this patient’s behavior?
a. Negative symptoms
b. Disorientation
c. Concrete thinking
d. Perseveration
e. Circumstantiality
32. A 32-year-old patient is being interviewed in his physician’s office. He responds to each
question, but he gives long answers with a great deal of tedious and unnecessary detail.
Which of the following symptoms best describes this patient’s presentation?

a. Blocking
b. Tangentiality
c. Circumstantiality
d. Looseness of associations
e. Flight of ideas

33. An 18-year-old man is brought to the emergency room by the police after he is found
walking along the edge of a high building. In the emergency room, he mumbles to himself
and appears to be responding to internal stimuli. When asked open-ended questions, he
suddenly stops his answer in the middle of a sentence, as if he has forgotten what to say.
Which of the following symptoms best describes this last behavior?

a. Incongruent affect
b. Blocking
c. Perseveration
d. Tangentiality
e. Thought insertion

34. A 26-year-old woman with panic disorder notes that during the middle of one of her attacks
she feels as if she is disconnected from the world, as though it were unreal or distant. Which
of the following terms best describes this symptom?

a. Dulled perception
b. Illusion
c. Retardation of thought
d. Depersonalization
e. Derealization

35. A patient with a chronic psychotic disorder is convinced that she has caused a recent
earthquake because she was bored and wishing for something exciting to occur. Which of the
following symptoms most closely describes this patient’s thoughts?

a. Thought broadcasting
b. Magical thinking
c. Echolalia
d. Nihilism
e. Obsession
36. A 43-year-old man tells his psychiatrist that he is spending several hours in the morning
checking all the light switches to make sure they are off. He states that if he does not do this,
he is overcome with anxiety. This is an example of which of the following symptoms?

a. Catalepsy
b. Compulsions
c. Magical thinking
d. Anhedonia
e. Folie a deux

37. A 45-year-old man with a chronic psychotic disorder is interviewed after being admitted to a
psychiatric unit. He mimics the examiner’s body posture and movements during the
interview. Which of the following terms best characterizes this patient’s symptom?
a. Folie a deux
b. Dereistic thinking
c. Echolalia
d. Echopraxia
e. Fugue

38. A 21 year student started appropriate treatment. One year later this woman returns to your
office with her mother for follow-up. Her symptoms remitted within a month. However, she
has not done well in her freshman year and for the past several months has continued to
experience worsening social isolation and amotivation. While she has not used any substances
since she last saw you, she reluctantly admits to occasionally hearing the devil
communicating with her. She tries to ignore the communication, and has taken to arranging
her books in a certain manner to prevent his controlling her thoughts. On her mental status
examination she makes poor eye contact and her affect is blunted. Her mother reports that
the patient now rarely calls home, though before she’d do so twice weekly. Which of the
following is the most likely diagnosis?
a. Schizophrenia
b. Delusional disorder
c. Bipolar disorder manic episode
d. Schizoaffective disorder

39. Which of the following is not typically associated with catatonia?

a. Mutism
b. Verbigeration
c. Stereotypes
d. Waxy flexibility

40. A 22-year-old single man is referred to you for a 1-year history of strange behavior characterized
by talking to the television, accusing local police of bugging his room, and carrying on
conversations with himself. His mother describes a 3-year history of progressive withdrawal from
social activities, and reports the patient dropped out of college and since has been living in his
room at home. Attempts to hold a job as a busboy at a local restaurant have abruptly ended
after disputes with the employers. What is the prevalence of this patient’s likely illness in the
general population?

a. 0.1%
b. 0.9% -1%
c. 2%
d. 5%
e. 0.2%

41. Which of the following best fits the definition “Loss of logical relations between thoughts”?

a. Neologism
b. Echolalia
c. Verbigeration
d. Clang association
e. Loosening of associations

42. Which of the following best fits the definition “Creation of a new expression or word ”?

a. Neologism
b. Echolalia
c. Verbigeration
d. Clang association
e. Loosening of associations

43. Which of the following best fits the definition “Repetition of interviewer’s words when
answering a question”?

a. Neologism
b. Echolalia
c. Verbigeration
d. Clang association
e. Loosening of associations

44. Which of the following best fits the definition “Words associated by sound rather than
meaning ”?

a. Neologism
b. Echolalia
c. Verbigeration
d. Clang association
e. Loosening of associations
45. Which of the following best fits the definition “Use of words in stereotypically repetitive
fashion”?

a. Neologism
b. Echolalia
c. Verbigeration
d. Clang association
e. Loosening of associations

46. Which of the following best fits the definition “Functional inhibition of speech”?

a. Echopraxia
b. Negativism
c. Anhedonia
d. Stereotypies
e. Mutism

47. Which of the following best fits the definition “Imitation of movements”?

a. Echopraxia
b. Negativism
c. Anhedonia
d. Stereotypies
e. Mutism

48. Which of the following best fits the definition “Repetitive, often bizarre, speech or behavior”?

a. Echopraxia
b. Negativism
c. Anhedonia
d. Stereotypies
e. Mutism

49. Which of the following best fits the definition “Unwillingness to cooperate without apparent
reason”?

a. Echopraxia
b. Negativism
c. Anhedonia
d. Stereotypies
e. Mutism
50. Which of the following best fits the definition “Diminution in ability to experience pleasure”?

a. Echopraxia
b. Negativism
c. Anhedonia
d. Stereotypies
e. Mutism

51. Which of the following delusion is most typical for delusional disorder?

a. Delusions of guilt
b. Intrusive, persistent, and uncontrollable thoughts or urges
c. Grandiose delusions
d. Bizarre delusions of being controlled
e. Delusions of jealousy

52. Which of the following delusion is most typical for Schizophrenia?

a. Delusions of guilt
b. Intrusive, persistent, and uncontrollable thoughts or urges
c. Grandiose delusions
d. Bizarre delusions of being controlled
e. Delusions of jealousy

53. Which of the following delusion is most typical for Manic episode?

a. Delusions of guilt
b. Intrusive, persistent, and uncontrollable thoughts or urges
c. Grandiose delusions
d. Bizarre delusions of being controlled
e. Delusions of jealousy

54. Which of the following delusion is most typical for Depressive disorder?

a. Delusions of guilt
b. Intrusive, persistent, and uncontrollable thoughts or urges
c. Grandiose delusions
d. Bizarre delusions of being controlled
e. Delusions of jealousy

55. Which of the following delusion is most typical for Obsessive Compulsive Disorder?

a. Delusions of guilt
b. Intrusive, persistent, and uncontrollable thoughts or urges
c. Grandiose delusions
d. Bizarre delusions of being controlled
e. Delusions of jealousy

56. Which of the following symptom is most typical for Body Dysmorphic Disorder?

a. Psychomotor retardation
b. Thought broadcasting
c. Easy distractibility with an elevated, expansive, or irritable mood
d. Nonbizarre persecutory or grandiose delusions
e. Preoccupation with an imagined or exaggerated defect in appearance

57. Which of the following symptom is most typical for Delusional disorder?

a. Psychomotor retardation
b. Thought broadcasting
c. Easy distractibility with an elevated, expansive, or irritable mood
d. Nonbizarre persecutory delusions
e. Preoccupation with an imagined or exaggerated defect in appearance

58. Which of the following symptom is most typical for Schizophrenia?

a. Psychomotor retardation
b. Thought broadcasting
c. Easy distractibility with an elevated, expansive, or irritable mood
d. Nonbizarre persecutory delusions
e. Preoccupation with an imagined or exaggerated defect in appearance

59. Which of the following symptom is most typical for Manic episode?

a. Psychomotor retardation
b. Thought broadcasting
c. Easy distractibility with an elevated, expansive, or irritable mood
d. Nonbizarre persecutory delusions
e. Preoccupation with an imagined or exaggerated defect in appearance

60. Which of the following symptom is most typical for Major depressive episode?

a. Psychomotor retardation
b. Thought broadcasting
c. Easy distractibility with an elevated, expansive, or irritable mood
d. Nonbizarre persecutory delusions
e. Preoccupation with an imagined or exaggerated defect in appearance

61. Which of the following best fits the statement “Patients believe that their family members
have been replaced by impostors”?

a. Capgras’s syndrome
b. Frégoli’s phenomenon
c. Clérambault’s syndrome
d. Delusional jealousy

62. Which of the following best fits the statement “Patients believe that different people are in
fact a single person who changes appearance or is in disguise”?

a. Capgras’s syndrome
b. Frégoli’s phenomenon
c. Clérambault’s syndrome
d. Delusional jealousy

63. Which of the following best fits the statement “False believe that someone (usually of higher
status or authority) is erotically attached to the patient”?

a. Capgras’s syndrome
b. Frégoli’s phenomenon
c. Clérambault’s syndrome
d. Delusion of doubles
e. Delusional jealousy

64. Which of the following best fits the statement “False believe about a spouse’s infidelity”?

a. Capgras’s syndrome
b. Frégoli’s phenomenon
c. Clérambault’s syndrome
d. Delusional jealousy

65. Which of the following best fits the definition “Mixture of random words”?

a. Loosening of associations
b. Flight of ideas
c. Word salad
d. Tangentiality
e. Circumstantiality
66. Which of the following best fits the definition “Quickly moving from one idea to another”?

a. Loosening of associations
b. Flight of ideas
c. Word salad
d. Tangentiality
e. Circumstantiality

67. Which of the following best fits the definition “Unnecessary details and irrelevant remarks
cause a delay in getting to the point”?

a. Loosening of associations
b. Flight of ideas
c. Word salad
d. Tangentiality
e. Circumstantiality

68. Which of the following best fits the definition “The patient seems always about to get near to
the matter in hand, but never quite reaches it”?

a. Loosening of associations
b. Flight of ideas
c. Word salad
d. Tangentiality
e. Circumstantiality

69. Which of the following best fits the definition “A belief that is firmly held against all
evidence to the contrary”?

a. Obsessions
b. Delusions
c. Made emotions
d. Thought broadcasting
e. Illusion

70. Which of the following best fits the definition “Compulsive preoccupation with a fixed idea”?

a. Obsessions
b. Delusions
c. Made emotions
d. Thought broadcasting
e. Illusion

71. Which of the following best fits the definition “Experiencing someone else's emotions”?
a. Obsessions
b. Delusions
c. Made emotions
d. Thought broadcasting
e. Illusion

72. A patient with severe depression developed a delusion of being wicked and deserving
punishment. Which of the following answer is correct?

a. Delusion is primary symptom


b. Depression is primary symptom
c. Both are primary symptoms
d. Both are secondary symptoms

73. Which of the following is not included in the disorders of the stream of thought?

a. Pressure of thought
b. Poverty of thought
c. Thought perseveration
d. Made thoughts
e. Thought blocking

74. Which of the following statement represents disadvantages of the DSM system?

a. The DSM system has substantially improved the reliability of diagnosis.


b. The DSM system may sacrifice validity for reliability.
c. The DSM system has clarified the diagnostic process and facilitated history taking.
d. DSM helps clinicians decide which symptoms must be present to rule in or to rule out a
particular diagnosis

75. Systems of classification for psychiatric diagnoses have several purposes. Which of the
following statement represents the ultimate purpose of classification ?

a. To distinguish one psychiatric diagnosis from another.


b. To provide a common language among health care professionals.
c. To explore the still unknown causes of many mental disorders.
d. To improve treatment and prevention efforts.

76. A 17-year-old boy is diagnosed with schizophrenia. What is the risk that one of his siblings
will develop the disease?
a. 2%
b. 5%
c. 10%
d. 20%
e. 30%

77. A 20-year-old woman is brought to the emergency room by her family because they have
been unable to get her to eat or drink anything for the past two days. The patient, although
awake, is completely unresponsive both vocally and nonverbally. She actively resists any
attempt to be moved. Her family reports that during the previous seven months she became
increasingly withdrawn, socially isolated, and bizarre, often speaking to people no one else
could see. Which of the following is the most likely diagnosis?

a. Schizoaffective disorder
b. Delusional disorder
c. Schizophreniform disorder
d. Catatonia

78. A 54-year-old man with a chronic mental illness seems to be constantly chewing. He does not
wear dentures. His tongue darts in and out of his mouth, and he occasionally smacks his lips.
He also grimaces, frowns, and blinks excessively. Which of the following disorders is most
likely in this patient?

a. Tourette’s syndrome
b. Akathisia
c. Tardive dyskinesia
d. Parkinson’s disease
e. Huntington’s disease

79. A 24-year-old woman comes to the emergency room with the chief complaint that “my
stomach is rotting out from the inside.” She states that for the last six months she has been
crying on a daily basis and that she has decreased concentration, energy, and interest in her
usual hobbies. She has lost 25 lb during that time. She cannot get to sleep, and when she does,
she wakes up early in the morning. For the past three weeks, she has become convinced that
she is dying of cancer and is rotting on the inside of her body. Also, in the past two weeks she
has been hearing a voice calling her name when no one is around. Which of the following is
the most likely diagnosis?

a. Delusional disorder
b. Schizoaffective disorder
c. Schizophreniform disorder
d. Schizophrenia
e. Major depression with psychotic features
80. A 19-year-old man is brought to the physician by his parents after he called them from
college, terrified that the Mafia was after him. He reports that he has eaten nothing for the
past six weeks other than canned beans because “they are into everything—I can’t be too
careful.” He is convinced that the Mafia has put cameras in his dormitory room and that they
are watching his every move. He occasionally hears the voices of two men talking about him
when no one is around. His roommate states that for the past two months the patient has
been increasingly withdrawn and suspicious. Which of the following is the most likely
diagnosis?

a. Delusional disorder
b. Schizoaffective disorder
c. Schizophreniform disorder
d. Schizophrenia
e. PCP intoxication

81. A 36-year-old woman is brought to the psychiatrist by her husband because for the past eight
months she has refused to go out of the house, believing that the neighbors are trying to harm
her. She is afraid that if they see her they will hurt her, and she finds many small bits of
evidence to support this. This evidence includes the neighbors’ leaving their garbage cans out
on the street to try to trip her, parking their cars in their driveways so they can hide behind
them and spy on her, and walking by her house to try to get a look into where she is hiding.
She states that her mood is fine and would be “better if they would leave me alone.” She
denies hearing the neighbors or anyone else talk to her, but is sure that they are out to “cause
her death and mayhem.” Which of the following is the most likely diagnosis?

a. Delusional disorder
b. Schizophreniform disorder
c. Schizoaffective disorder
d. Schizophrenia
e. Major depression with psychotic features

82. A 35-year-old woman has lived in a state psychiatric hospital for thepast 10 years. She spends
most of her day rocking, muttering softly to herself, or looking at her reflection in a small
mirror. She needs help with dressing and showering, and she often giggles and laughs for no
apparent reason. Which of the following is the most likely diagnosis?

a. Schizophrenia
b. Delusional disorder
c. Bipolar disorder, manic phase
d. Schizoaffective disorder
e. Schizophreniform disorder
83. A 21-year-old man is brought to the emergency room by his parents because he has not slept,
bathed, or eaten in the past three days. The parents report that for the past six months their
son has been acting strangely and “not himself.” They state that he has been locking himself
in his room, talking to himself, and writing on the walls. Six weeks prior to the emergency
room visit, their son became convinced that a fellow student was stealing his thoughts and
making him unable to learn his school material. In the past two weeks, they have noticed that
their son has become depressed and has stopped taking care of himself, including bathing,
eating, and getting dressed. On exam, the patient is dirty, disheveled, and crying. He
complains of not being able to concentrate, a low energy level, and feeling suicidal. Which of
the following is the most likely diagnosis for this patient?
a. Schizoaffective disorder
b. Schizophrenia
c. Bipolar I disorder
d. Schizoid personality disorder

84. A 26-year-old woman was brought to the emergency room by her husband. He states that for
the past four days she has been increasingly agitated, not sleeping, eating little, and “talking
nonsense.” On examination, the patient’s speech is notable in that she cannot seem to stick to
one topic. When asked a question, she answers with a stream of sentences that seem to be
vaguely related to the others, but very rapidly veer off topic. Which of the following
statements best describes the finding on this patient’s examination?

a. She has a disorder of thought content


b. She has flight of ideas
c. She is delusional
d. She has ideas of reference
e. She has an obsession

85. A 47-year-old woman is brought to the emergency room after she jumped off an overpass in a
suicide attempt. In the emergency room she states that she wanted to kill herself because the
devil had been tormenting her for many years. After stabilization of her fractures, she is
admitted to the psychiatric unit, where she is treated with risperidone and sertraline. After
two weeks she is no longer suicidal and her mood is euthymic. However, she still believes
that the devil is recruiting people to try to persecute her. In the past 10 years, the patient has
had three similar episodes prior to this one. Throughout this time, she has never stopped
believing that the devil is persecuting her. Which of the following is the most appropriate
diagnosis for this patient?

a. Delusional disorder
b. Schizoaffective disorder
c. Schizophrenia, paranoid type
d. Schizophreniform disorder
e. Major depression with psychotic features
86. A 40-year-old woman is arrested by the police after she is found crawling through the
window of a movie star’s home. She states that the movie star invited her into his home
because the two are secretly married and “it just wouldn’t be good for his career if everyone
knew.” The movie star denies the two have ever met, but notes that the woman has sent him
hundreds of letters over the past two years. The woman has never been in trouble before and
lives an otherwise isolated and unremarkable life. Which of the following is the most likely
diagnosis?

a. Delusional disorder
b. Schizoaffective disorder
c. Bipolar I disorder
d. Cyclothymia
e. Schizophreniform disorder

87. A 25-year-old woman is treated with an antipsychotic medication after being admitted to an
inpatient unit for a psychotic break. Six days after being started on the medication, she is
noted to be repeatedly pacing the halls. Even when she is seated, she jiggles her legs and taps
her hands on the table constantly. When asked about this behavior, she replies that “my arms
and legs just feel like they have to move.” Which of the following is the most likely
explanation for this patient’s behavior?

a. She is experiencing a side effect from the medication


b. She is becoming increasingly psychotic
c. She is experiencing withdrawal from an illicit substance
d. She is fearful of the inpatient environment
e. She is experiencing the onset of neuroleptic malignant syndrome

88. Which of the following statements about delusions is true?

a. They are found almost exclusively in schizophrenia


b. Grandiose delusions are rarely encountered except in mania
c. They involve a disturbance of thought content
d. They involve a disturbance in perception
e. They are a type of hallucination

89. A 21-year-old man is brought to the emergency room by his parents


after he attempted to stab himself with a knife. They note that during the
past eight months the patient has become increasingly isolated, disheveled,
and bizarre. For the last three days he has locked himself in his room,
apparently afraid that someone will hurt him. When questioned, the
patient states that he needs to “cut his heart out” because a voice is telling
him that is the only way he will end up in heaven. What is the percentage
of patients with this diagnosis who ultimately complete such suicide
attempts?
a. 1%
b. 5%
c. 10%
d. 20%
e. 30

90. A 26-year-old woman is brought to the emergency room by her husband after she begins
screaming that her children are calling to her and becomes hysterical. The husband states that
two weeks previously, the couple’s two children were killed in a car accident, and since that
time the patient has been agitated, disorganized, and incoherent. He states that she will not
eat because she believes he has been poisoning her food, and she has not slept for the past two
days. The patient believes that the nurses in the emergency room are going to cause her harm
as well. The patient is sedated and later sent home. One week later, all her symptoms remit
spontaneously. Which of the following is the most likely diagnosis for this patient?

a. Delirium
b. Schizophreniform disorder
c. Major depression with psychotic features
d. Brief psychotic disorder
e. Posttraumatic stress disorder

91. Two days after delivering a healthy, full-term baby girl, a 25-year-old woman becomes
acutely agitated and disoriented. She refuses to feed her baby, stating that the baby was borne
of the devil. She hears voices telling her to drown her daughter if she wants to save her soul.
Which of the following statements is correct?

a. The woman is having a full-blown episode of schizophrenia


b. The baby should be removed by a child protection agency immediately
c. The woman should be warned never to have another child
d. The woman’s disorder is most closely related to schizophrenia
e. The woman should be treated with antipsychotic and antidepressant medications

92. A 28-year-old woman is brought to see a psychiatrist by her mother. The patient insists that
nothing is wrong with her, but the mother notes that the patient has been slowly but
progressively isolating herself from everyone. She now rarely leaves the house. The mother
says she can hear the patient talking to “people who aren’t there” while she’s in her room. On
examination, the patient is noted to have auditory hallucinations and the delusional belief
that her mother is going to kick her out of the house so that it can be turned into a theme
park. Which of the following is the lifetime prevalence for this disorder?

a. 1%
b. 3%
c. 5%
d. 10%
e. 15%

93. A 25-year-old woman is diagnosed with schizophrenia when, after the sudden death of her
mother, she begins complaining about hearing the voice of the devil and is suddenly afraid
that other people are out to hurt her. Her history indicates that she has also experienced a
three-year period of slowly worsening social withdrawal, apathy, and bizarre behavior. Her
family history includes major depression in her father. Which of the following details of her
history leads the physician to suspect that her outcome may be poor?

a. She is female
b. She was age 25 at diagnosis
c. She had an acute precipitating factor before she began hearing voices
d. She had an insidious onset of her illness
e. There is a history of affective disorder in her family

94. Studies of the relationship between gender and schizophrenia have


generally demonstrated which of the following facts?

a. Age of onset for females is usually earlier than for males


b. Females tend to have a worse prognosis than males
c. The impairment in male patients is, on average, greater than in female patients
d. Females respond more poorly to medication than do males
e. Female monozygotic twins have a higher concordance rate for the disease than
do male monozygotic twins

95. Families of patients with schizophrenia who are overtly hostile and
overly controlling affect the patient in what way?

a. Increased relapse rate


b. Decreased rate of compliance
c. High likelihood that this behavior led to the patient’s first break of the disease
d. Increased likelihood that the patient’s schizophrenia will be of the paranoid
type
e. Decreased risk of suicidal behavior

96. A patient reports to you that for the past week or two he has had the belief that his intestines
and his heart have been removed. When asked about his lack of getting out in the world, he
responds “What world? There is no world!” This aspect of the patient’s illness would best be
referred to as which of the following?
a. schizoaffective disorder
b. Capgras syndrome
c. folie à deux
d. Cotard syndrome
e. major depression
97. All of the following are associated with a good prognosis in schizophrenia except
a. Sudden onset of symptoms
b. No affective symptoms
c. Confusion during psychosis
d. Severe precipitating stressor
e. Few premorbid schizoid traits.

98. Tactile hallucinations, sometimes called haptic hallucinations and are frequently experienced
together with olfactory hallucinations. Which of the following is correct regarding this
statement?
a. True
b. False

99. Reliability refers to the capacity to predict prognosis and outcome, response to treatment,
and ultimately etiology. Which of the following is correct regarding this statement?
a. True
b. False
100. Term neurocognitive disorder is widely used and often preferred for conditions
affecting younger individuals, such as impairment secondary to traumatic brain injury or HIV
infection. Which of the following is correct regarding this statement?
a. True
b. False

101. Avolition includes reductions in the expression of emotions in the face, intonation of
speech and movements of the hand/head, Which of the following is correct regarding this
statement?

a. True
b. False

102. Which of the following is the lifetime prevalence for delusional disorder disorder?

a. 1%
b. 3%
c. 0.2%
d. 10%
e. 15%
103. Manic episode require the presence of an abnormally elevated, expansive, or irritable
mood lasting at least 4 day and present most of the day, nearly every day. Which of the
following is correct regarding this statement?

a. True
b. False

104. The disturbance in mood and the change in functioning in hypomanic episode are
observable by others. Which of the following is correct regarding this statement?

c. True
a. False

105. A 32-year-old woman is diagnosed with major depression. What is the chance that
her identical twin sister will develop the same disease?

a. 5%
b. 20%
c. 50%
d. 70%
e. 90%

106. A 27-year-old woman comes to a psychiatrist with the chief complaint of feeling
depressed her entire life. While she states that she has never been so depressed that she was
unable to function, she never feels really good for more than a week or two at a time. She has
never been suicidal or psychotic, though her self-esteem is chronically low. Which of the
following is the most likely diagnosis?

a. Major depression
b. Adjustment disorder
c. Cyclothymia
d. Bipolar disorder
e. Dysthymia

107. A 25-year-old man comes to the psychiatrist with a chief complaintof depressed mood
for one month. His mother, to whom he was very close, died one month ago, and since that
time he has felt sad and been very tearful. He has difficulty concentrating, has lost 3 lb, and is
not sleeping soundly through the night. Which of the following is the most likely diagnosis?

a. Major depression
b. Dysthymia
c. Posttraumatic stress disorder
d. Adjustment disorder
e. Uncomplicated bereavement
108. A 32-year-old woman is brought to the emergency room by thepolice after she was
found standing in the middle of a busy highway, naked, commanding the traffic to stop. In
the emergency room she is agitated and restless, with pressured speech and an affect that
alternates between euphoric and irritable. Her father is contacted and states that this kind of
behavior runs in the family. Which of the following is the most likely diagnosis?

a. Delirium
b. Cyclothymia
c. Schizophrenia
d. Bipolar disorder, manic
e. Bipolar disorder
f. mixed state

109. A 52-year-old man comes to the physician with the chief complaint of feeling
depressed for the past two months. He notes that he is not sleeping well, has lost 25 lb in the
last six weeks, and is experiencing anergia and anhedonia. In addition, in the past four weeks
he has begun to hear the voice of his dead father telling him that he is a failure and has begun
worrying that his organs are rotting away. Which of the following statements is true?

a. The patient should be started on an SSRI and an antipsychotic


b. The patient is having an acute schizophrenic episode
c. The patient is likely suffering from a factitious disorder with psychological
symptoms
d. The patient is likely abusing alcohol
e. The patient should be started on an SSRI alone

110. A 26-year-old man comes to the physician with the chief complaint of a depressed
mood for the past five weeks. He has been feeling down, with decreased concentration,
energy, and interest in his usual hobbies. Six weeks prior to this office visit he had been to the
emergency room for an acute asthma attack and was started on prednisone. Which of the
following is the most likely diagnosis?

a. Mood disorder secondary to a general medical condition


b. Substance-induced mood disorder
c. Major depression
d. Adjustment disorder
e. Dysthymia

111. What is the lifetime risk for suicide in patients with mood disorders?

a. 1 to 3%
b. 3 to 5%
c. 10 to 15%
d. 20 to 30%
e. 30 to 40%

112. What is the prevalence of alcoholism among patients with a history of major
depression?
a. 5%
b. 10%
c. 20%
d. 35%
e. 60%

113. A 29-year-old woman comes to the emergency room because she has been feeling
depressed for the past three weeks. She complains of an inability to sleep, anergia, anhedonia,
loss of libido, and weight loss of 10 lb. When she is asked about suicidal ideation, she is
evasive and refuses to give an answer, stating, “If I talk to you about suicidal ideation, you
will put me in the hospital, even if I deny it.” She notes that she was hospitalized once
previously for a suicide attempt in which she had slashed her wrist. Which of the following
elements of this patient’s story indicates to the physician that she is at a higher risk for
suicide?

a. Her age
b. Her gender
c. Her previous suicide attempt
d. Her difficulty sleeping
e. The method used in her last suicide attempt

114. An elderly man has been profoundly depressed for several weeks. He cries easily and
is intensely preoccupied with trivial episodes from his past, which he considers unforgivable
sins. This patient awakens every morning at 3 A.M. and cannot go back to sleep. Anything his
family has tried to cheer him up has failed. He has completely lost his appetite and appears
gaunt and emaciated. Which of the following is the most likely diagnosis?
a. Atypical depression
b. Double depression
c. Cyclothymic disorder
d. Major depression
e. Schizoaffective disorder
f. Seasonal affective disorder
115. A young woman who has felt mildly unhappy and dissatisfied with herself for most of
her life has been severely depressed, irritable, and anhedonic for three weeks. Which of the
following is the most likely diagnosis?
a. Atypical depression
b. Double depression
c. Cyclothymic disorder
d. Major depression
e. Schizoaffective disorder
f. Seasonal affective disorder

116. Since he moved to Maine from his native Florida three years earlier, a college student
has had great difficulty preparing for the winter-term courses. He starts craving sweets and
feeling sluggish, fatigued, and irritable in late October. These symptoms worsen gradually
during the following months, and by February he has consistently gained several pounds. His
mood and energy level start improving in March, and by May he is back to baseline. Which
of the following is the most likely diagnosis?

a. Atypical depression
b. Double depression
c. Cyclothymic disorder
d. Major depression
e. Schizoaffective disorder
f. Seasonal derpessive disorder

117. A middle-aged man is chronically preoccupied with his health. For many years he
feared that his irregular bowel functions meant he had cancer. Now he is very worried about
having a serious heart disease, despite his physician’s assurance that the occasional “extra
beats” he detects when he checks his pulse are completely benign. Which of the following is
the most likely diagnosis?

a. Somatic symptom disorder


b. Illness anxiety disorder
c. Delusional disorder
d. Pain disorder
e. Conversion disorder

118. A 23-year-old woman arrives at the emergency room complaining that, out of the
blue, she had been seized by an overwhelming fear, associated with shortness of breath and a
pounding heart. These symptoms lasted for approximately 20 minutes, and while she was
experiencing them, she feared that she was dying or going crazy. The patient has had four
similar episodes during the past month, and she has been worrying that they will continue to
recur. Which of the following is the most likely diagnosis?
a. Acute psychotic episode
b. Hypochondriasis
c. Panic disorder
d. Generalized anxiety disorder
e. Posttraumatic stress disorder

119. Which of the following doesn’t refer to Bleuler’s fundamental symptoms in


schizophrenia?

a. Association loosening
b. Avolition
c. Thought broadcasting
d. Affective blunting

120. Which of the following refers to Schneider's “symptoms of first rank”?


a. Voices repeating
b. Two or more hallucinatory voices
c. Voices commenting
d. Thought broadcasting
e. all of the above

121. Which of the following refers to positive symptoms of schizophrenia?


a. Alogia
b. Hallucination
c. Affective blunting
d. Avolition
e. Anhedonia

122. Which of the following refers to negative symptoms of schizophrenia?


a. Hallucinations
b. Delusions
c. Disorganized Speech
d. Affective blunting

123. Which of the following refers to non bizarre delusion?


a. Thought withdrawal
b. Thought insertion
c. Belief that one is under surveillance by the police
d. Delusions of control

124. Which of the following is not typically associated with catatonia?


a. Stupor
b. Catalepsy
c. Waxy flexibility
d. Blunted affect
e. Mutism
f. Negativism

125. Which of the following is not typically associated with disturbance of perception?

a. Illusion
b. Thought insertion
c. Hallucination
d. Preidolic illusion

126. Which of the following is not typically associated with disturbance of the stream of
thought?

a. Pressure of thought
b. Poverty of thought
c. Thought perseveration
d. Thought blocking
e. Made impulses

127. Which of the following is not included in the DSM 5 chapter “Schizophrenia
Spectrum and Other Psychotic Disorder”?

a. Schizotypal (Personality) Disorder


b. Delusional Disorder
c. Paranoid personality disorder
d. Brief Psychotic Disorder
e. Schizophreniform Disorder
f. Schizophrenia
g. Schizoaffective Disorder

128. Which of the following types of delusion is more common in manic episode?

a. Erotomaniac delusion
b. Grandiose delusion
c. Jealous type
d. Persecutory delusion
e. Somatic delusion

129. Which of the following types of delusion is more common in depressive episode?

a. Erotomaniac type
b. Grandiose type
c. Delusion of guilty
d. Persecutory type
e. Somatic type
130. Which of the following statement is correct regarding Jealous type of delusion?

a. Central theme of the delusion involves the individual’s belief that he or she is being conspired
against, cheated or spied
b. The central theme of the delusion involves bodily functions or sensations.
c. The central theme of the delusion is that person’s spouse or lover is unfaithful
d. The central theme of the delusion is that another person is in love with the individual

131. Which of the following is not included in the DSM 5 chapter “Bipolar and related
disorders”?

a. Dysthymia
b. Bipolar I disorder
c. Bipolar II disorder
d. Cyclothymic disorder
e. Substance/medication-induced bipolar and related disorder

132. After witnessing a violent argument between her parents, a young woman develops
sudden blindness but does not appear as distraught as would be expected by this
development. Her pupils react normally to light, and she manages to somehow avoid
obstacles when walking. Her parents, who are in the middle of a bitter divorce, put aside
their differences to focus on their daughter’s illness. Which of the following is the most likely
diagnosis?

a. Factitious disorder
b. Malingering
c. Somatization disorder
d. Conversion disorder
e. Histrionic personality disorder

133. A 28-year-old taxi driver is chronically consumed by fears of having accidentally run
over a pedestrian. Although he tries to convince himself that his worries are silly, his anxiety
continues to mount until he drives back to the scene of the “accident” and proves to himself
that nobody lies hurt in the street. This behavior is an example of which of the following?

a. A compulsion secondary to an obsession


b. An obsession triggered by a compulsion
c. A delusional ideation
d. A typical manifestation of obsessive-compulsive personality disorder
e. A phobia
134. An 18-year-old girl is brought to the psychiatrist by her mother because the girl takes
showers that last two to three hours and she is unable to stop the behavior, in spite of the fact
that she would like to do so. On questioning, the patient states that she feels “dirty” and
“contaminated” by the germs in the house, and if she does not shower, she feels increasingly
anxious. Once in the shower, she must use a specific technique for washing, and if she
deviates from it, she must start over. In discussing the diagnosis of the problems with the
patient, which of the following statements should be made to her?

a. It is unusual to find this disorder in one so young


b. The disorder is usually quite responsive to pharmacological intervention
c. The disorder is usually caused by traumatic events
d. The disorder is passed on to one’s children at rates of 25 to 30%
e. The disorder has frequent spontaneous periods of complete remission

135. A young woman, who has a very limited memory of her childhood years but knows
that she was removed from her parents because of their abuse and neglect, frequently cannot
account for hours or even days of her life. She hears voices that alternately plead, reprimand,
or simply comment on what she is doing. Occasionally, she does not remember how and
when she arrived at a specific location. She finds clothes she does not like in her closet, and
she does not remember having bought them. Her friends are puzzled because sometimes she
acts in a childish, dependent way and at other times becomes uncharacteristically aggressive
and controlling. These symptoms are commonly seen in which of the following disorders?

a. Dissociative amnesia
b. Depersonalization disorder
c. Dissociative identity disorder
d. Schizophrenia

136. A 45-year-old woman was physically and sexually assaulted in her own house by two
intruders. She cannot remember anything about the incident. She is concerned about the
memory loss because she prides herself on her excellent memory, and she would like to
remember the faces of the two intruders so that she can help identify them to the police.
Which of the following statements should be made to this patient?

a. It is likely that she will experience future episodes of memory loss now that she
has incurred this one
b. Most cases revert spontaneously
c. It is likely that she will develop a major depressive episode once she remembers
the incident
d. Antidepressants will help reverse the memory loss
e. The loss of memory is usually irreversible

137. A 34-year-old secretary climbs 12 flights of stairs every day to reach her office
because she is terrified by the thought of being trapped in the elevator. She has never had any
traumatic event occur in an elevator; nonetheless, she has been terrified of them since
childhood. Which of the following is the most likely diagnosis?

a. Social phobia
b. Performance anxiety
c. Generalized anxiety disorder
d. Specific phobia
e. Agoraphobia

138. A 23-year-old woman presents to her physician with the chief complaint that she is
anxious about the way she looks. She notes that for “as long as she can remember” she has
been obsessed about the fact that something must be wrong with her face. She notes that her
eyes are too far apart and her nose is misshapen. She states that this concern is “ruining her
life” because she spends all her time isolated from others so that they cannot see her face. The
physician did not notice anything unusual about the patient’s face, but the patient cannot be
consoled by this statement. Which of the following is the most likely diagnosis?

a. Body dysmorphic disorder


b. Delusional disorder
c. Obsessive-compulsive disorder
d. Somatization disorder
e. Hypochondriasis

139. A young executive is periodically required to give reports of his department’s progress
in front of the firm’s CEO. Although usually confident and well prepared, the young man
becomes very anxious prior to each presentation. Once he is in front of his audience, he
experiences dry mouth, heart palpitations, and profuse sweating. Which of the following
statements concerning this disorder is correct?

a. Females have a higher prevalence


b. Onset in adolescence is rare
c. Medications are not effective treatment options
d. Cognitive-behavioral therapy has been proven to be effective
e. Heredity does not play a role

140. During the hectic weeks preceding her wedding, a 22-year-old woman in good health
and without a previous history of psychiatric illness occasionally feels unreal and detached
from her own body, “like in a dream.” The episodes last a few minutes and resolve
spontaneously. Which of the following statements is correct?

a. These symptoms can occur in people without psychiatric disorders


b. The patient suffers from depersonalization disorder
c. Depersonalization symptoms are rare in other psychiatric disorders
d. The patient suffers from somatization disorder
e. The patient is malingering
141. A 45-year-old policeman who has demonstrated great courage on more than one
occasion while on duty is terrified of needles. Which of the following is the most likely
diagnosis?

a. Agoraphobia
b. Panic disorder
c. Obsessive-compulsive disorder
d. Social phobia
e. Adjustment disorder
f. Specific phobia
g. Acute stress disorder

142. For several months, a 32-year-old housewife has been unable to leave her house
unaccompanied. When she tries to go out alone, she is overwhelmed by anxiety and fear that
something terrible will happen to her and nobody will be there to help. Which of the
following is the most likely diagnosis?

a. Agoraphobia
b. Panic disorder
c. Obsessive-compulsive disorder
d. Social phobia
e. Adjustment disorder
f. Specific phobia
g. Acute stress disorder

143. A 17-year-old girl blushes, stammers, and feels completely foolish when one of her
classmates or a teacher asks her a question. She sits at the back of the class hoping not to be
noticed because she is convinced that the other students think she is unattractive and stupid.
Which of the following is the most likely diagnosis?

a. Agoraphobia
b. Panic disorder
c. Obsessive-compulsive disorder
d. Social phobia
e. Adjustment disorder
f. Specific phobia
g. Acute stress disorder

144. Two years after she was saved from her burning house, a 32-year-old woman
continues to be distressed by recurrent dreams and intrusive thoughts about the event.
Which of the following is the most likely diagnosis?
a. Somatic symptom disorder
b. Specific phobia
c. Dissociative identity disorder
d. Obsessive-compulsive disorder
e. Dissociative fugue
f. Posttraumatic stress disorder
g. Body dysmorphic disorder
h. Dysthymia

145. A 20-year-old student is very distressed by a small deviation of his nasal septum. He is
convinced that this minor imperfection is disfiguring, although others barely notice it. Which
of the following is the most likely diagnosis?

a. Somatic symptom disorder


b. Specific phobia
c. Dissociative identity disorder
d. Obsessive-compulsive disorder
e. Dissociative fugue
f. Posttraumatic stress disorder
g. Body dysmorphic disorder
h. Dysthymia

146. A nun is found in a distant city working in a cabaret. She is unable to remember
anything about her previous life. Which of the following is the most likely diagnosis?

a. Somatic symptom disorder


b. Specific phobia
c. Dissociative identity disorder
d. Obsessive-compulsive disorder
e. Dissociative fugue
f. Posttraumatic stress disorder
g. Body dysmorphic disorder
h. Dysthymia

147. A 35-year-old woman is often late to work because she must shower and dress in a
very particular order or else she becomes increasingly anxious. Which of the following is the
most likely diagnosis?

a. Somatic symptom disorder


b. Specific phobia
c. Dissociative identity disorder
d. Obsessive-compulsive disorder
e. Dissociative fugue
f. Posttraumatic stress disorder
g. Body dysmorphic disorder
h. Dysthymia

148. For the past three years, a 24-year-old college student has suffered from chronic
headaches, fatigue, shortness of breath, dizziness, ringing ears, and constipation. He is
incensed when his primary physician recommends a psychiatric evaluation because no
organic cause for his symptoms could be found. Which of the following is the most likely
diagnosis?
a. Somatic symptom disorder
b. Specific phobia
c. Dissociative identity disorder
d. Obsessive-compulsive disorder
e. Dissociative fugue
f. Posttraumatic stress disorder
g. Body dysmorphic disorder
h. Dysthymia

149. Which of the following doesn’t refer to the psychological symptoms of anxiety?
a. Irritability
b. Sensitivity to noise
c. Difficulty in swallowing
d. Restlessness
e. Poor concentration
f. Worrying thoughts

150. Which of the following is not included in the DSM 5 chapter “Anxiety disorder”?
a. Panic disorder
b. Generlized anxiety disorder
c. Obsessive compulsive disorder
d. adjustment disorder

151. A 47-year-old man with a master’s degree in chemistry lives alone in a halfway house
and subsists on panhandling and collecting redeemable cans. Ten years ago he lost his job in a
large firm because he was found to have repeatedly stolen company money and used it to bet
on horse racing. Afterward, he had several other jobs but always lost them because he stole
money. He also stole and borrowed money from friends and relatives. When asked about this
behavior, the patient stated that he felt very guilty about it but “couldn’t seem to stop”
himself. Which of the following diagnoses best fits this patient’s symptoms?

a. Antisocial personality disorder


b. Conduct disorder
c. Pathological gambling
d. Fugue state
e. Factitious disorder with psychological symptoms

152. A 65-year-old woman lives alone in a dilapidated house, although her family
members have tried in vain to move her to a better dwelling. She wears odd and out-of-
fashion clothes and rummages in the garbage cans of her neighbors to look for redeemable
cans and bottles. She is very suspicious of her neighbors. She was convinced that her
neighbors were plotting against her life for a brief time after she was mugged and thrown
onto the pavement by a teenager, but now thinks that this is not the case. She believes in the
“power of crystals to protect me” and has them strewn haphazardly throughout her house.
Which of the following is the most likely diagnosis?

a. Autism
b. Schizophrenia, paranoid type
c. Schizotypal personality disorder
d. Avoidant personality disorder
e. Schizoid personality disorder

153. A demanding 25-year-old woman begins psychotherapy stating that she is both
desperate and bored. She reports that for the past five or six years she has experienced
periodic anxiety and depression and has made several suicidal gestures. She also reports a
variety of impulsive and selfdefeating behaviors and sexual promiscuity. She wonders if she
might be a lesbian, though most of her sexual experiences have been with men. She has
abruptly terminated two previous attempts at psychotherapy. In both cases she was enraged
at the therapist because he was unwilling to prescribe anxiolytic medications. Which of the
following is the most likely diagnosis?

a. Dysthymia
b. Histrionic personality disorder
c. Antisocial personality disorder
d. Borderline personality disorder
e. Impulse control disorder not otherwise specified

154. A 28-year-old woman begins seeing a psychiatrist because, she says, “I am just so very
lonely.” Her speech is excessively impressionistic and lacks specific detail. She flirts
constantly with the physician and is “hurt” when the therapist does not notice her new
clothes or hairstyle. Which of the following personality disorders is most likely?

a. Paranoid
b. Schizotypal
c. Schizoid
d. Narcissistic
e. Borderline
f. Histrionic
g. Antisocial
h. Obsessive-compulsive

155. A 42-year-old man comes to the psychiatrist at the insistence of his boss because he
constantly misses important deadlines. The man states that everyone at work is lazy and that
no one lives up to his own standards for perfection. He is angry when the physician starts the
interview three minutes later than the appointed time. He notes that he is always fighting
with his wife because he is a “pack rat” and is unable to throw anything out. During the
interview, he appears very rigid and stubborn. Which of the following personality disorders is
most likely?

a. Paranoid
b. Schizotypal
c. Schizoid
d. Narcissistic
e. Borderline
f. Histrionic
g. Antisocial
h. Obsessive-compulsive

156. A 34-year-old woman comes to the psychiatrist on the advice of her mother, because
the patient still lives at home and will not make any decisions without her mother’s
reassurance. The patient’s mother accompanies the patient to the appointment. She states that
the patient becomes anxious when her mother must leave the home because the patient is
terrified that her mother will die and the patient will have to take care of herself, something
she feels incapable of doing. Which of the following personality disorders is most likely?

a. Paranoid
b. Schizotypal
c. Schizoid
d. Narcissistic
e. Borderline
f. Histrionic
g. Antisocial
h. Obsessive-compulsive
i. Dependent

157. A 25-year-old high school dropout has been arrested more than 12 times for various
assault, fraud, and attempted murder charges. He has been in many physical fights, usually
after he got caught cheating at cards. On examination, he seems relaxed and even cocky, and
he shows no remorse for his actions. Which of the following personality disorders is most
likely

a. Paranoid
b. Schizotypal
c. Schizoid
d. Narcissistic
e. Borderline
f. Histrionic
g. Antisocial
h. Obsessive-compulsive

158. Estimates of the lifetime prevalence rates of all personality disorders


in the general population are in what range?

a. 0 to 1%
b. 1 to 2%
c. 3 to 5%
d. 10 to 20%
e. 20 to 30%

159. A 35-year-old man comes to the psychiatrist at the insistence of his wife. She reports
that for as long as she has known her husband, he has been exceedingly suspicious of those
around him. She states that he trusts no one and is always convinced that friends and
neighbors are trying to take advantage of him in some way. Once slighted, he holds onto
grudges for a very long time. The patient is guarded and mildly hostile toward the physician,
convinced that he is not interested but, rather, is just “earning too much money” from the
encounter. Which of the following is true of patients with this disorder?

a. They usually also suffer from paranoid psychosis


b. They have a predisposition to develop schizophrenia
c. They often have a preoccupation with helping the weak and the powerless
d. They usually present themselves in a quiet and humble fashion
e. They are often litigious

160. A young librarian has been exceedingly shy and fearful of people since childhood. She
longs to make friends, but even casual social interactions cause her a great deal of shame and
anxiety. She has never been at a party, and she has requested to work in the least active
section of her library, even though this means lower pay. She cannot look at her rare
customers without blushing, and she is convinced that they see her as incompetent and
clumsy. Which of the following personality disorders is most likely?

a. Schizotypal
b. Avoidant
c. Dependent
d. Schizoid
e. Paranoid

161. An off-Broadway actor consistently bores his friends and acquaintances by talking
incessantly about his exceptional talent and his success on the stage. He does not seem to
realize that other people do not share his high opinion of his acting talent and are not
interested in his monologues. When a director criticizes the way he delivers his lines during
rehearsal, the actor goes into a rage and accuses the director of trying to jeopardize his career
out of jealousy. Which personality disorder represents the most likely diagnosis?

a. Histrionic
b. Narcissistic
c. Borderline
d. Paranoid
e. Antisocial

162. A 24-year-old woman drops out of college after two weeks. When asked why, she
states that although she would desperately like to have friends, she is afraid to approach
anyone because “they would think I’m just a nerd.” Furthermore, in the middle of a class, one
of the professors asked her a question and she became extremely uncomfortable. She has
never had a significant relationship with anyone other than her parents and sister.
Match this behavior with the most appropriate personality disorder.

a. Paranoid
b. Schizotypal
c. Schizoid
d. Narcissistic
e. Borderline
f. Histrionic
g. Antisocial
h. Obsessive-compulsive
i. Dependent
j. Avoidant

163. A 32-year-old man comes to the psychiatrist because he is anxious about his new job.
He notes that he previously held a job shelving books in the back of a library, but due to
budget cuts he has been forced to interact with customers. He states he doesn’t like being
around people and prefers being by himself. He appears emotionally cold and detached
during the interview. Match this behavior with the most appropriate personality disorder.
a. Paranoid
b. Schizotypal
c. Schizoid
d. Narcissistic
e. Borderline
f. Histrionic
g. Antisocial
h. Obsessive-compulsive

164. Which of the following refer to the disorders of the stream of thought?
f. Circumstantiality
g. clang associations
h. Word salad
i. Flight of ideas
j. All of the above

165. Which of the following best fits the definition of Stereotypy?

f. Temporary loss of muscle tone and weakness precipitated by a variety of emotional states
g. Pathological imitation of movements of one person by another
h. Ingrained, habitual involuntary movement
i. Repetitive fixed pattern of physical action or speech
j. Subjective feeling

166. Which of the following doesn’t characterize anhedonia?


f. Difficulty describing or being aware of emotions
g. Derealization
h. Loss of pleasure
i. Inability to experience normal emotions
j. Withdrawal from interest

167. Which of the following doesn’t characterize psychomotor retardation?

f. Paucity of spontaneous movements


g. Reduced speech amplitude and flow
h. Indecisiveness
i. Poor concentration
j. Restlessness

168. Which of the following statement best characterizes double depression?

f. Two episodes of major depressive disorder per month consistently


g. Superimposed bipolar II disorder and atypical depression
h. Recurrent major depressive disorder superimposed with dysthymic disorder
i. Two family members suffering from major depressive disorder concurrently
j. Recurrent major depressive disorder with current symptoms twice as disabling as usual

169. Which of the following increase the risk of developing anxiety disorders?

f. Eating disorders
g. Depression
h. Substance abuse
i. Allergies
j. All of the above

170. Which of the following examples or situations is most likely to cause PTSD?

f. Involvement in an earthquake
g. Being diagnosed with cancer
h. Rape
i. Witnessing a crime
j. Observing a flood

171. Unexpected panic attacks are required to make which of the following diagnosis?

f. panic disorder
g. social phobia
h. specific phobia
i. generalized anxiety disorder
j. all of the above

172. Which of the following best fits the definition of anxiety disorder?

f. An emotional state identified by panic attacks.


g. An emotional condition classified by excessive checking.
h. Disordered thinking.
i. An excessive or aroused state characterized by feelings of apprehension, uncertainty and fear.

173. You are treating a 48-year-old married female on the inpatient medical unit for
pyelonephritis; she has responded well to appropriate antibiotic therapy and has been afebrile
for the last 24 hours. You inform her of likely discharge if she continues to improve. The next
morning, however, she complains of feeling feverish and achy, and having dysuria again. The
nursing staff reports that she has a sudden fever of 103°F. You treat the fever with
acetaminophen and perform a physical examination, order chest x-rays, draw blood, and
order a urinalysis with culture. While you are awaiting these results, the nurse informs you
that she witnessed the patient dipping her thermometer into a hot cup of tea before her
temperature was taken. Which of the following is the most likely diagnosis?

f. Factitious disorder
g. Functional neurological symptom disorder
h. Illness anxiety disorder
i. Malingering
j. Somatic symptom disorder

174. Which of the following doesn’t refer to the negative symptoms of schizophrenia?

f. alogia
g. affective flattening
h. avolition
i. aggressivity
j. inattentiveness

175. Which of the following statement best fits Dysthymic Disorder?

f. Usually begins in adulthood


g. Does not respond to anti-depressants
h. Symptoms present most of the days for at least 2 years
i. Is usually limited to one or two episodes
j. Is characteristically marked by psychomotor agitation or psychomotor retardation

176. Which of the following statements is true with regard to factitious disorder?

f. It is synonymous with malingering


g. The patient’s goal is to assume and maintain the sick role
h. The patient’s goal is to avoid unpleasant consequences or work
i. Patients visit their PCP’s office often but rarely are hospitalized
j. It is easily diagnosed

177. Which of the following doesn’t present the goal of the psychiatric interview?
f. Assessment the nature of the problem
g. Demonstration to the patient psychiatrist’s expertise
h. Establishment of a therapeutic relationship
i. Development of a treatment plan

178. When ending a psychiatric interview, the doctor should

f. End the interview on time


g. Leave unfinished business for the next appointment
h. Give the patient the opportunity to ask questions
179. A psychiatric patient who, move from thought to thought that relate in some way,
but never gets to the point has a disturbance in the form of thought called
f. Word salad
g. Circumstantiality
h. Tangentiality
i. Verbigeration
j. Blocking

180. Perceptual disturbances include all of the following except


f. Hallucinations
g. Hypnagogic experiences
h. Echolalia
i. Illusion

181. Mr. A is a 22-year-old law student living alone in the school’s dormitory. He had
hopes to become a federal judge. Over the last 8 months his academic performance has
declined, and he is being considered for academic probation. He has become increasingly
isolated and withdrawn, and the girl he was dating broke off their relationship. He believes
that she had been replaced by a look-alike from a distant planet, and that his fellow law
students are conspiring against him. He believes that they snort and sneeze whenever he
enters the classroom. He reports getting distracting “signals” from the television set, and that
he hears voices of “the devil” calling to him. He called his father and asked for his help.
Distressed, his father, himself a lawyer, brought him to the psychiatric emergency room for
an evaluation. Physical examination was normal, and laboratory tests, including head
computed tomography (CT) scan and urine toxicology, were negative. Which of the following
psychiatric conditions is the most likely diagnosis?

f. Brief psychotic disorder


g. Schizophreniform disorder
h. Delusional disorder
i. Schizophrenia
j. Malingering

182. Appropriate psychosocial therapies in the management and treatment of


schizophrenia include

f. Social skills training


g. Case management
h. Individual psychotherapy
i. Group therapy
j. All of the above
183. Features weighing toward a good prognosis in schizophrenia include all of the
following except

f. Affective symptoms
g. Strong family support
h. Undifferentiated or disorganized features
i. Acute onset

184. Possible risk factors for the development of schizophrenia include

f. Birth during winter months


g. Increased number of birth complications
h. Social class
i. Poverty
j. Recent immigration status
k. All of the above

185. Which of the following is not part of the DSM-5 criteria for diagnosing a Brief
Psychiatric disorder?

e. Presence of positive psychotic symptoms


f. Grossly abnormal psychomotor behavior
g. Sudden onset
h. The psychotic symptoms occur exclusively during a full major depressive episode
i. Disturbance lasts at least 1 day but less than 1 month

186. Which of the following statements regarding mood disorders is false?

e. Approximately 15 percent of depressed patients will eventually commit suicide.


f. Depressive disorders are more common in women
g. A rapid-cycling pattern is associated with a good prognosis.
h. Manic forms of mood disorders predominate in men
i. 1 out of 4 patients with an acute depressive episode will have recurrences throughout life

187. Which of the following statements regarding hypomanic episodes is false?

f. It is characterized by mild elevations of mood


g. Patients often experience increased energy levels
h. It is ego-syntonic
i. It often progresses to manic psychosis
j. It can be mobilized by antidepressant use
188. Which of the following is not a component of the DSM 5 diagnostic criteria for
OCD?
e. Obsessions are acknowledged as excessive or unreasonable
f. There are attempts to ignore or suppress compulsive thoughts or impulses
g. Obsessions or compulsions are time consuming, and take more than 1 hour a day
h. The person recognized the obsessional thoughts as a product of outside themselves

189. Specific phobias are defined as:

f. Excessive worry bouts triggered by a specific object or situation.


g. An abnormal sensitivity to light.
h. An excessive, unreasonable, persistent fear triggered by a specific object or situation.
i. A persistent fear of social situations.

190. It is considered that successful CBT treatments of social phobia include elements of
the following:

f. Exposure therapy.
g. Social skills training.
h. Cognitive restructuring.
i. All of the above.

191. Which of the following physical symptoms are associated with Panic attacks:

f. Heart palpitations.
g. Perspiring.
h. Hyperventilating.
i. All of the above.

192. Generalised Anxiety Disorder (GAD) is a pervasive condition in which the sufferer
experiences:
f. Fear of fear.
g. Continual apprehension and anxiety about future events.
h. Continual flashbacks to past events.
i. A desire to check that the environment is safe.

193. Which of the following is considered to be a symptom of Post Traumatic Stress


Disorder (PTSD):
f. Increased arousal.
g. Avoidance and numbing of emotions.
h. Re-experiencing.
i. All of the above.

194. Which of the following is a diagnostic criterion for anorexia nervosa in DSM 5?
f. A refusal to maintain a minimal body weight
g. A pathological fear of gaining weight
h. A distorted body image in which, even when clearly emaciated, sufferers continue to insist
they are overweight
i. All of the above

195. Individuals with bulimia have a perceived lack of control over their eating behaviour,
and often report which of the following?

f. High levels of self-disgust


g. Low self-esteem
h. High levels of depression
i. All of the above

196. Which of the following is a common treatment for bulimia?


e. Social skill Therapy
f. Cognitive Behaviour Therapy
g. Psychodynamic therapy
h. Humanistic therapy

197. The negative symptoms of schizophrenia include all of the following except
f. Alogia
g. Affective flattening
h. Avolition
i. Hallucination
j. Inattentiveness

198. Which of the following best fits definition “fear of dirt and germs”?

e. Acrophobia
f. Cynophobia
g. Mysophobia
h. Xenophobia
i. Ailurophobia

199. Which of the following best fits definition “ Fear of heights”?


e. Acrophobia
f. Cynophobia
g. Mysophobia
h. Xenophobia
i. Ailurophobia

200. Which of the following best fits definition “Fear of strangers”?


f. Acrophobia
g. Cynophobia
h. Mysophobia
i. Xenophobia
j. Ailurophobia

201. Which of the following best fits definition “Fear of dogs”?


f. Acrophobia
g. Cynophobia
h. Mysophobia
i. Xenophobia

202. Which of the following best fits definition “Fear of snakes”?


f. Acrophobia
g. Cynophobia
h. Ophidiophobia
i. Mysophobia
j. Xenophobia

203. Which of the following statement is not true with regard of schizophrenia?
c. Schizophrenia is associated with significant social and occupational dysfunction
d. maintaining employment are frequently impaired by avolition
e. Most individuals are employed at a lower level than in general population
f. Most patients , particularly men, are married and have not limited social contacts

204. Which of the following statement is not true with regard of risk of suicide in
schizophrenia?

c. About 20% attempt suicide on one or more occasions, and many more have significant
suicidal ideation.
d. Schizophrenia is associated with significant social and occupational dysfunction
e. Females are at higher risk then males
f. Suicide risk remains high over the whole lifespan for males and females
g. Suicidal behavior is sometimes in response to command hallucinations to harm oneself

205. Which of the following is a diagnostic criterion for delusional disorder in DSM 5?

c. The presence of one or more delusions with a duration of 1 month or longer


d. The functioning is not markedly impaired
e. behavior is not obviously bizarre or odd
f. All of the above

206. Which of the following statement is closely associated with delusional disorder?
c. global function is generally better than that observed in schizophrenia
d. The essential feature of delusional disorder is a sudden onset
e. Patients may have rapid shifts from one intense affect to another.
f. The lifetime prevalence of delusional disorder has been estimated at around 0.9%

207. Criterion A for schizoaffective disorder requires an uninterrupted period of illness


during which Criterion A of schizophrenia is met. Which of the following additional
symptoms must be present to fulfill diagnostic criteria for schizoaffective disorder?
d. An anxiety episode
e. A major depressive or manic episode
f. Dysthymia
g. Cyclothymia

208. In which of the following disorders can psychotic symptoms occur?


b. Bipolar and depressive disorder
c. Schizophrenia
d. Delusional Diorder
e. PTSD
f. All of the above

209. Which of the following is not commonly recognized type of delusion?

g. Persecutory
h. Erotomanic
i. Alien Abduction
j. Somatic
k. Grandiose
210. Schizophrenia spectrum and other psychotic disorder are defined by abnormalities in
one or more of five domains, four of which are also considered psychotic symptoms. Which
of the following is not considered a psychotic symptom?
f. Delusions
g. Hallucinations
h. Disorganized thinking
i. Disorganized or abnormal motor behavior
j. Avolition
211. Approximately what percentage of individuals who experience a single episode of
mania will go on to have recurrent mood episodes?
f. 90%
g. 50%
h. 25%
i. 10%
j. 1%

212. Which of the following is more common in men with bipolar I disorder Than in
women with the disorder?
e. Rapid cycling
f. Alcohol abuse
g. Eating disorder
h. Anxiety disorder

213. Which of the following features confers a worse prognosis for a patient with bipolar
II disorder?
e. Younger age
f. Higher educational level
g. Rapid-cycling pattern
h. “Married” marital status
i. Less severe depressive episodes

214. The course of bipolar II disorder would likely to worse for individuals wo have an
onset of the disorder at which of the following age?
g. Age 10 years
h. Age 20 years
i. Age 40 years
j. Age 70 years

215. Which of the following disorders is included in the “Anxiety Disorder” chapter of
DSM 5?
e. OCD
f. PTSD
g. Acute stress disorder
h. Generalized anxiety disorder

216. A 32 old man has regularly experienced panic attacks when out of his home alone and
when on the bus. He now avoids leaving home for fear of experiencing these attacks. What is
the most appropriate diagnosis?
f. Panic disorder
g. Agoraphobia
h. Specific phobia
i. Two separate disorders: Panic disorder and Agoraphobia
j. Delusional disorder

217. Although onset of specific phobia can occur at any age ,it most typically develops at
which age?

h. childhood
i. Late adolescence
j. Middle age
k. Old age

218. In social phobia the object of an individual’s fear is the potential for which of
following?
f. Social or occupational impairment
g. Harm to self or others
h. Embarrassment
i. Separation of objects of attachment

219. Social phobia differs from normative shyness in that the disorder leads to which of
the following?
f. Social or occupational dysfunction
g. Marked social reticence
h. Avoidance of social situations
i. Derealization or depersonalization

220. In what aspect of generalized anxiety disorder do men and women most commonly
differ?

e. Course
f. Symptom profile
g. Degree of impairment
h. Patterns of comorbidity

221. Which of the following disorders is included in the “Obsessive compulsive disorder”
chapter of DSM 5?
f. Body dimorphic disorder
g. Specific phobia
h. Eating disorder
i. Pathological gambling

222. Which of the following is a diagnostic criterion for OCD in DSM 5?


g. Presence of obsessions and/or compulsions
h. The obsessions or compulsions are time-consuming
i. The obsessions or compulsions cause clinically significant distress or impairment in social,
occupational, or other important areas of functioning
j. All of the above

223. Criterion A for OCD requires presence of obsessions, compulsions, or both. Which of
the following additional symptoms must be present to fulfill diagnostic criteria for OCD?
e. An anxiety episode
f. The obsessions or compulsions take more than 1 hour per day or cause clinically significant
distress
g. A major depressive or manic episode
h. Dysthymia

224. Criterion B for acute stress disorder requires the presence of nine or more symptoms
from five categories. Which of the following is not one of these five categories?
a. Intrusion
b. Dissociation
c. Confusion
d. Avoidance
e. Arrousal

225. Which of the following stressful situations would meet criterion A for the diagnosis
of Acute stress disorder?
a. finding out that one’s spouse has been fired
b. Failing an important final examination
c. Receiving a serious medical diagnosis
d. Being in the cross of a police shootout but not being harmed

226. Criterion A for body dysmorphic disorder requires preoccupation with one or more
perceived defects or flaws in physical appearance that are not observable or appear slight to
others. Which of the following additional symptoms must be present to fulfill diagnostic
criteria for OCD?
a. An anxiety episode
b. The symptoms are time consuming
c. The individual has performed repetitive behaviors or mental acts
d. Major depressive episode

227. Which of the following disorders is included in the “trauma and stressor related
disorder” chapter of DSM 5?

a. Reactive attachment disorder,


b. posttraumatic stress disorder (PTSD),
c. acute stress disorder,
d. adjustment disorder
e. all of the above

228. Which of the following experiences would not qualify as exposure to trauma in the
acute stress disorder or PTSD?
a. Hearing that one’s brother was killed in combat
b. Hearing that one’s close friend was paralyzed in motor accident
c. Hearing that one’s child has been kidnapped
d. Hearing that one’s company had suddenly closed

229. How many criterion B symptoms are required to be present for the diagnosis of acute
stress disorder?
a. One
b. Three
c. Five
d. Seven
e. Nine

230. Dissociative disorders involve disruptions or discontinuities in the operations and


integration of many areas of psychological functioning. Which of the following is not a
functional area affected in dissociative disorder?
a. Memory
b. Consciousness
c. Perception
d. Delusional believes
e. Emotional responses

231. Which of the following disorders is not included in the “dissociative disorder” chapter
of DSM 5?

a. dissociative identity disorder,


b. dissociative amnesia,
c. depersonalization/derealization disorder,
d. conversion disorder

232. Criterion A for dissociative disorder requires disruption of and/or discontinuity in the
normal integration of consciousness, memory, identity, emotion, perception, body
representation, motor control, and behavior. Which of the following additional symptoms
must be present to fulfill diagnostic criteria DD?

a. An anxiety episode
b. The symptoms are time consuming
c. The individual has performed repetitive behaviors or mental acts
d. Major depressive episode
e. Recurrent gaps in the recall of everyday events, important personal information, and/
or traumatic events

233. Which of the following is a diagnostic criterion for dissociative identity disorder?

a. The presence of two or more distinct personality states or an experience of possession


b. Sudden alterations or discontinuities in sense of self
c. Recurrent dissociative amnesia
d. Clinically significant distress or impairment
e. All of the above

234. Criterion A for dissociative identity disorder requires presence of two or more distinct
personality states or an experience of possession. Which of the following additional
symptoms must be present to fulfill diagnostic criteria of DID?

a. The identities that arise during possession are not a normal part of a broadly accepted
cultural or religious practice
b. An anxiety episode
c. The symptoms are time consuming
d. The individual has performed repetitive behaviors or mental acts
e. Major depressive episode

235. Which of the following best fits the definition “Apparently purposeful travel or
bewildered wandering that is associated with amnesia for identity”?
a. Dissociative amnesia
b. Dissociative fugue
c. conversion disorder
d. Dissociative identity disorder

236. Which of the following best fits the definition “a failure to recall events during a
circumscribed period of time”?
a. Generalized amnesia
b. Systematized amnesia
c. Localized amnesia
d. Continuous amnesia

237. Which of the following best fits the definition “the individual loses memory for a
specific category of information (e.g., all memories relating to one's family, a particular
person, or childhood sexual abuse)”?
a. Generalized amnesia
b. Systematized amnesia
c. Localized amnesia
d. Continuous amnesia

238. Criterion A for derealization depersonalization disorder requires experiences of


unreality, detachment, or being an outside observer with respect to one’s self or surroundings.
Which of the following additional symptoms must be present to fulfill diagnostic criteria of
derealization depersonalization disorder?
a. The identities that arise during possession are not a normal part of a broadly accepted
cultural or religious practice
b. During the depersonalization or derealization experiences, reality testing remains intact
c. The symptoms are time consuming
d. The individual has performed repetitive behaviors or mental acts
e. Major depressive episode

239. Dissociative amnesia most often involves which of the following types of amnesia?

a. Generalized amnesia
b. Systematized amnesia
c. Localized or selective amnesia
d. Continuous amnesia

240. What diagnosis would apply to patient with high level of anxiety about having a
disease and many associated somatic symptoms?
a. Hypochondriasis
b. Illness anxiety disorder
c. Somatic symptom disorder
d. Generalized anxiety disorder
e. Specific phobia

241. Which of the following disorders is not included in the “somatic symptom disorder”
chapter of DSM 5?

a. Somatic symptom disorder,


b. Illness anxiety disorder,
c. Conversion disorder
d. Factitious disorder
e. Malingering

242. Criterion A for somatic symptom disorder requires one or more somatic symptoms
that are distressing or result in significant disruption of daily life. Which of the following
additional symptoms must be present to fulfill diagnostic criteria of somatic symptom
disorder?

a. An anxiety episode
b. The individual has performed repetitive behaviors or mental acts
c. Major depressive episode
d. The state of being symptomatic is persistent (typically more than 6 months)
e. Selective amnesia
243. What is the essential diagnosis feature for factitious disorder?
a. Somatic symptoms
b. Conscious misrepresentation and deception
c. External gain associated with illness
d. Absence of another medical disorder that may cause the symptoms

244. Criterion A for conversion disorder requires one or more symptoms of altered
voluntary motor or sensory function. Which of the following additional symptoms must be
present to fulfill diagnostic criteria of conversion disorder?

a. Clinical findings provide evidence of incompatibility between the symptom and


recognized neurological or medical conditions
b. The individual has performed repetitive behaviors or mental acts
c. Major depressive episode
d. The state of being symptomatic is persistent (typically more than 6 months)
e. Selective amnesia

245. What is the minimum average frequency of binge eating required for
diagnosis od DSM 5?

a. Once weekly for the last 3 months


b. Once weekly for the last 4 months
c. Every other week for the last 3 months
d. Every other week for the last 4 months

246. What are the two subtypes of anorexia nervosa?


a. Restricting type and binge eating/purging type
b. Energy sparing type and binge eating/purging type
c. Low calorie type and restricting type
d. Restricting type and low weight type

247. Both major and mild neurocognitive disorders can increase the risk of
delirium. Traditionally delirium distinguished from dementia on the basis of the key
features: acute onset, impairment in attention and which of the following?
a. Fluctuating course
b. Steady course
c. Presence of mania
d. Presence of depression

248. Which of the following disorders is not included in the neurocognitive disorder
chapter of DSM 5?
a. Delirium
b. Major neurocognitive disorder
c. Mild neurocognitive disorder
d. Amnesic disorder

249. Which of the following cognitive or perceptual disturbances are associated with
borderline personality disorder?

a. Odd thinking and speech


b. Ideas of reference
c. Odd believes
d. Transient stress related paranoid ideations
e. compulsions

250. Which of the following behavior or states would be highly unusual in an individual
with schizoid personality disorder?
a. An angry outburst at a colleague who criticizes his work
b. Turning down invitation to the party
c. Lacking desire for sexual experiences
d. Difficulty working in a collaborative working enviroment
251. Which of the following disorders is not included in the neurocognitive disorder
chapter of DSM 5?
k. Delirium
l. Major neurocognitive disorder
m. Mild neurocognitive disorder
n. Amnesic disorder

252. Which of the following cognitive or perceptual disturbances are associated with
borderline personality disorder?

k. Odd thinking and speech


l. Ideas of reference
m. Odd believes
n. Transient stress related paranoid ideations
o. compulsions

253. Which of the following behavior or states would be highly unusual in an


individual with schizoid personality disorder?
k. An angry outburst at a colleague who criticizes his work
l. Turning down invitation to the party
m. Lacking desire for sexual experiences
n. Difficulty working in a collaborative working enviroment

254. What diagnosis would apply to patient with high level of anxiety about having a
disease and many associated somatic symptoms?
k. Hypochondriasis
l. Illness anxiety disorder
m. Somatic symptom disorder
n. Generalized anxiety disorder
o. Specific phobia

255. Which of the following disorders is not included in the “somatic symptom
disorder” chapter of DSM 5?

k. Somatic symptom disorder,


l. Illness anxiety disorder,
m. Conversion disorder
n. Factitious disorder
o. Malingering

256. What is the essential diagnosis feature for factitious disorder?


k. Somatic symptoms
l. Conscious misrepresentation and deception
m. External gain associated with illness
n. Absence of another medical disorder that may cause the symptoms

257. Criterion A for conversion disorder requires one or more symptoms of altered
voluntary motor or sensory function. Which of the following additional symptoms
must be present to fulfill diagnostic criteria of conversion disorder?

k. Clinical findings provide evidence of incompatibility between the symptom and


recognized neurological or medical conditions
l. The individual has performed repetitive behaviors or mental acts
m. Major depressive episode
n. The state of being symptomatic is persistent (typically more than 6 months)
o. Selective amnesia

258. Both major and mild neurocognitive disorders can increase the risk of delirium.
Traditionally delirium distinguished from dementia on the basis of the key features:
acute onset, impairment in attention and which of the following?
k. Fluctuating course
l. Steady course
m. Presence of mania
n. Presence of depression

259. In what aspect of generalized anxiety disorder do men and women most
commonly differ?

j. Course
k. Symptom profile
l. Degree of impairment
m. Patterns of comorbidity

260. Which of the following disorders is included in the “Obsessive compulsive


disorder” chapter of DSM 5?
k. Body dimorphic disorder
l. Specific phobia
m. Eating disorder
n. Pathological gambling

261. Which of the following is a diagnostic criterion for OCD in DSM 5?


k. Presence of obsessions and/or compulsions
l. The obsessions or compulsions are time-consuming
m. The obsessions or compulsions cause clinically significant distress or impairment in
social, occupational, or other important areas of functioning
n. All of the above

262. Criterion A for OCD requires presence of obsessions, compulsions, or both.


Which of the following additional symptoms must be present to fulfill diagnostic
criteria for OCD?
k. An anxiety episode
l. The obsessions or compulsions take more than 1 hour per day or cause clinically
significant distress
m. A major depressive or manic episode
n. Dysthymia

263. Criterion B for acute stress disorder requires the presence of nine or more
symptoms from five categories. Which of the following is not one of these five
categories?
k. Intrusion
l. Dissociation
m. Confusion
n. Avoidance
o. Arrousal

264. Which of the following stressful situations would meet criterion A for the
diagnosis of Acute stress disorder?
j. finding out that one’s spouse has been fired
k. Failing an important final examination
l. Receiving a serious medical diagnosis
m. Being in the cross of a police shootout but not being harmed

265. Criterion A for body dysmorphic disorder requires preoccupation with one or
more perceived defects or flaws in physical appearance that are not observable or
appear slight to others. Which of the following additional symptoms must be present to
fulfill diagnostic criteria for OCD?
i. An anxiety episode
j. The symptoms are time consuming
k. The individual has performed repetitive behaviors or mental acts
l. Major depressive episode

266. Which of the following disorders is included in the “trauma and stressor related
disorder” chapter of DSM 5?

k. Reactive attachment disorder,


l. posttraumatic stress disorder (PTSD),
m. acute stress disorder,
n. adjustment disorder
o. all of the above

267. Which of the following experiences would not qualify as exposure to trauma in
the acute stress disorder or PTSD?
j. Hearing that one’s brother was killed in combat
k. Hearing that one’s close friend was paralyzed in motor accident
l. Hearing that one’s child has been kidnapped
m. Hearing that one’s company had suddenly closed

268. How many criterion B symptoms are required to be present for the diagnosis of
acute stress disorder?
k. One
l. Three
m. Five
n. Seven
o. Nine

269. Dissociative disorders involve disruptions or discontinuities in the operations


and integration of many areas of psychological functioning. Which of the following is
not a functional area affected in dissociative disorder?
k. Memory
l. Consciousness
m. Perception
n. Delusional believes
o. Emotional responses
270. Which of the following disorders is not included in the “dissociative disorder”
chapter of DSM 5?

j. dissociative identity disorder,


k. dissociative amnesia,
l. depersonalization/derealization disorder,
m. conversion disorder

271. Criterion A for dissociative disorder requires disruption of and/or discontinuity


in the normal integration of consciousness, memory, identity, emotion, perception,
body representation, motor control, and behavior. Which of the following additional
symptoms must be present to fulfill diagnostic criteria DD?

l. An anxiety episode
m. The symptoms are time consuming
n. The individual has performed repetitive behaviors or mental acts
o. Major depressive episode
p. Recurrent gaps in the recall of everyday events, important personal information, and/
or traumatic events

272. Which of the following is a diagnostic criterion for dissociative identity


disorder?

j. The presence of two or more distinct personality states or an experience of possession


k. Sudden alterations or discontinuities in sense of self
l. Recurrent dissociative amnesia
m. Clinically significant distress or impairment
n. All of the above

273. Criterion A for dissociative identity disorder requires presence of two or more
distinct personality states or an experience of possession. Which of the following
additional symptoms must be present to fulfill diagnostic criteria of DID?

j. The identities that arise during possession are not a normal part of a broadly accepted
cultural or religious practice
k. An anxiety episode
l. The symptoms are time consuming
m. The individual has performed repetitive behaviors or mental acts
n. Major depressive episode
274. Which of the following best fits the definition “Apparently purposeful travel or
bewildered wandering that is associated with amnesia for identity”?
k. Dissociative amnesia
l. Dissociative fugue
m. conversion disorder
n. Dissociative identity disorder

275. Which of the following best fits the definition “a failure to recall events during
a circumscribed period of time”?
i. Generalized amnesia
j. Systematized amnesia
k. Localized amnesia
l. Continuous amnesia

276. Which of the following best fits definition “fear of dirt and germs”?

j. Acrophobia
k. Cynophobia
l. Mysophobia
m. Xenophobia
n. Ailurophobia

277. Which of the following statement is not true with regard of schizophrenia?
j. Schizophrenia is associated with significant social and occupational dysfunction
k. maintaining employment are frequently impaired by avolition
l. Most individuals are employed at a lower level than in general population
m. Most patients , particularly men, are married and have not limited social contacts

j. Which of the following statement is not true with regard of risk of suicide in
schizophrenia?

j. About 20% attempt suicide on one or more occasions, and many more have significant
suicidal ideation.
k. Schizophrenia is associated with significant social and occupational dysfunction
l. Females are at higher risk then males
m. Suicide risk remains high over the whole lifespan for males and females
n. Suicidal behavior is sometimes in response to command hallucinations to harm oneself
278. Which of the following is a diagnostic criterion for delusional disorder in DSM
5?

j. The presence of one or more delusions with a duration of 1 month or longer


k. The functioning is not markedly impaired
l. behavior is not obviously bizarre or odd
m. All of the above

279. Which of the following statement is closely associated with delusional


disorder?
j. global function is generally better than that observed in schizophrenia
k. The essential feature of delusional disorder is a sudden onset
l. Patients may have rapid shifts from one intense affect to another.
m. The lifetime prevalence of delusional disorder has been estimated at around 0.9%

280. Criterion A for schizoaffective disorder requires an uninterrupted period of


illness during which Criterion A of schizophrenia is met. Which of the following
additional symptoms must be present to fulfill diagnostic criteria for schizoaffective
disorder?
j. An anxiety episode
k. A major depressive or manic episode
l. Dysthymia
m. Cyclothymia

281. In which of the following disorders can psychotic symptoms occur?


i. Bipolar and depressive disorder
j. Schizophrenia
k. Delusional Diorder
l. PTSD
m. All of the above

282. Which of the following is not commonly recognized type of delusion?

k. Persecutory
l. Erotomanic
m. Alien Abduction
n. Somatic
o. Grandiose
283. A 35-year-old man comes to the psychiatrist at the insistence of his wife. She
reports that for as long as she has known her husband, he has been exceedingly
suspicious of those around him. She states that he trusts no one and is always convinced
that friends and neighbors are trying to take advantage of him in some way. Once
slighted, he holds onto grudges for a very long time. The patient is guarded and mildly
hostile toward the physician, convinced that he is not interested but, rather, is just
“earning too much money” from the encounter. Which of the following is true of
patients with this disorder?

a. They usually also suffer from paranoid psychosis


b. They have a predisposition to develop schizophrenia
c. They often have a preoccupation with helping the weak and the powerless
d. They usually present themselves in a quiet and humble fashion
e. They are often litigious

284. A young librarian has been exceedingly shy and fearful of people since
childhood. She longs to make friends, but even casual social interactions cause her a
great deal of shame and anxiety. She has never been at a party, and she has requested to
work in the least active section of her library, even though this means lower pay. She
cannot look at her rare customers without blushing, and she is convinced that they see
her as incompetent and clumsy. Which of the following personality disorders is most
likely?

a. Schizotypal
b. Avoidant
c. Dependent
d. Schizoid
e. Paranoid

285. An off-Broadway actor consistently bores his friends and acquaintances by


talking incessantly about his exceptional talent and his success on the stage. He does
not seem to realize that other people do not share his high opinion of his acting talent
and are not interested in his monologues. When a director criticizes the way he
delivers his lines during rehearsal, the actor goes into a rage and accuses the director of
trying to jeopardize his career out of jealousy. Which personality disorder represents
the most likely diagnosis?

a. Histrionic
b. Narcissistic
c. Borderline
d. Paranoid
e. Antisocial

286. Which of the following doesn’t characterize psychomotor retardation?

j. Paucity of spontaneous movements


k. Reduced speech amplitude and flow
l. Indecisiveness
m. Poor concentration
n. Restlessness

287. Which of the following statement best characterizes double depression?

j. Two episodes of major depressive disorder per month consistently


k. Superimposed bipolar II disorder and atypical depression
l. Recurrent major depressive disorder superimposed with dysthymic disorder
m. Two family members suffering from major depressive disorder concurrently
n. Recurrent major depressive disorder with current symptoms twice as disabling as usual

288. Which of the following examples or situations is most likely to cause PTSD?

k. Involvement in an earthquake
l. Being diagnosed with cancer
m. Rape
n. Witnessing a crime
o. Observing a flood

289. Unexpected panic attacks are required to make which of the following
diagnosis?

j. panic disorder
k. social phobia
l. specific phobia
m. generalized anxiety disorder
n. all of the above

290. Which of the following best fits the definition of anxiety disorder?
k. An emotional state identified by panic attacks.
l. An emotional condition classified by excessive checking.
m. Disordered thinking.
n. An excessive or aroused state characterized by feelings of apprehension, uncertainty
and fear.

291. You are treating a 48-year-old married female on the inpatient medical unit for
pyelonephritis; she has responded well to appropriate antibiotic therapy and has been
afebrile for the last 24 hours. You inform her of likely discharge if she continues to
improve. The next morning, however, she complains of feeling feverish and achy, and
having dysuria again. The nursing staff reports that she has a sudden fever of 103°F.
You treat the fever with acetaminophen and perform a physical examination, order
chest x-rays, draw blood, and order a urinalysis with culture. While you are awaiting
these results, the nurse informs you that she witnessed the patient dipping her
thermometer into a hot cup of tea before her temperature was taken. Which of the
following is the most likely diagnosis?

g. Factitious disorder
h. Functional neurological symptom disorder
i. Illness anxiety disorder
j. Malingering
k. Somatic symptom disorder

292. Which of the following statement best fits Dysthymic Disorder?

h. Usually begins in adulthood


i. Does not respond to anti-depressants
j. Symptoms present most of the days for at least 2 years
k. Is usually limited to one or two episodes
l. Is characteristically marked by psychomotor agitation or psychomotor retardation

293. Which of the following statements is true with regard to factitious disorder?

g. It is synonymous with malingering


h. The patient’s goal is to assume and maintain the sick role
i. The patient’s goal is to avoid unpleasant consequences or work
j. Patients visit their PCP’s office often but rarely are hospitalized
k. It is easily diagnosed

294. Which of the following doesn’t present the goal of the psychiatric interview?
g. Assessment the nature of the problem
h. Demonstration to the patient psychiatrist’s expertise
i. Establishment of a therapeutic relationship
j. Development of a treatment plan

295. When ending a psychiatric interview, the doctor should

h. End the interview on time


i. Leave unfinished business for the next appointment
j. Give the patient the opportunity to ask questions

Each of the questions or incomplete statements below is followed by several suggested responses or
completions. Select the one that is best in each case.

296. A 23-year-old woman comes to the emergency room with the chief complaint that she
has been hearing voices for seven months. Besides the hallucinations, she has the idea that the
radio is giving her special messages. When asked the meaning of the proverb “People in glass
houses should not throw stones,” the patient replies, “Because the windows would break.”
Which of the following mental status findings does this patient display?

o. Poverty of content

p. Concrete thinking

q. Flight of ideas

r. Loose associations

s. Autistic thinking

297. A 24-year-old man is admitted to the inpatient psychiatry unit after his mother observed
him standing in place for hours at a time in abnormal postures. During his exam, the patient
stands with one arm raised directly above his head and the other straight out in front of him. He
is mute, does not appear aware of his surroundings, and actively resists any attempts to change
his position. Which of the following best describes the patient’s behavior?

p. Apraxia

q. Dystonia

r. Synesthesia
s. Catatonia

t. Trance state

298. Formal thought disorders include

p. circumstantiality

q. clang associations

r. neologisms

s. flight of ideas

t. all of the above

299. Stereotypy is

o. temporary loss of muscle tone and weakness precipitated by a variety of emotional states

p. pathological imitation of movements of one person by another

q. ingrained, habitual involuntary movement

r. repetitive fixed pattern of physical action or speech

s. subjective feeling

300. A 56-year-old man is brought to the physician’s office by his wife because she has noted
a personality change during the past three months. While the patient is being interviewed, he
answers every question with the same three words. Which of the following symptoms best fits
this patient’s behavior

p. Negative symptoms

q. Disorientation

r. Concrete thinking

s. Perseveration
301. A 69-year-old man is brought to see his physician by his wife. She notes that over the
past year he has experienced a slow, stepwise decline in his cognitive functioning. One year ago
she felt his thinking was “as good as it always had been,” but now he gets lost around the house
and can’t remember simple directions. The patient insists that he feels fine, though he is
depressed about his loss of memory. He is eating and sleeping well. Which of the following is the
most likely diagnosis?

o. Multi-infarct dementia

p. Mood disorder secondary to a general medical condition

q. Schizoaffective disorder

r. Delirium

s. Major depression

302. A psychiatric resident is called to consult on the case of a 75-year-old woman who had
undergone a hip replacement two days before. On examination, the resident notes that the
patient states the date as 1956, and she thinks she is at her son’s house. These impairments
illustrate which aspect of the mental status examination?

o. Concentration

p. Memory

q. Thought process

r. Orientation

s. Level of consciousness

303. The negative symptoms of schizophrenia include all of the following except

p. alogia

q. affective flattening

r. avolition

s. aggressivity
t. inattentiveness

304. Erotomania, the delusional disorder in which the person makes repeated efforts to
contact the object of the delusion, through letter, phone call, and stalking, is also referred to as

o. Cotard’s syndrome

p. Clérambault’s syndrome

q. Fregoli’s syndrome

r. Ganser’s syndrome

s. Capgras’s syndrome

305. A patient with a chronic psychotic disorder is convinced that she has caused a recent
earthquake because she was bored and wishing for something exciting to occur. Which of the
following symptoms most closely describes this patient’s thoughts?

n. Thought broadcasting

o. Magical thinking

p. Echolalia

q. Nihilism

r. Obsession

306. A 52-year-old man is sent to see a psychiatrist after he is disciplined at his job because
he consistently turns in his assignments late. He insists that he is not about to turn in anything
until it is “perfect, unlike all of my colleagues.” He has few friends because he annoys them with
his demands for “precise timeliness” and because of his lack of emotional warmth. This has been
a lifelong pattern for the patient, though he refuses to believe the problems have anything to do
with his personal behavior. Which of the following is the most likely diagnosis for this patient?

o. Obsessive-compulsive disorder
p. Obsessive-compulsive personality disorder

q. Borderline personality disorder

r. Bipolar disorder, mixed state

s. Anxiety disorder not otherwise specified

307. A 56-year-old man has been hospitalized for a myocardial infarction. Two days after
admission, he awakens in the middle of the night and screams that there is a man standing by
the window in his room. When the nurse enters the room and turns on a light, the patient is
relieved to learn that the “man” was actually a drape by the window. This misperception of
reality is best described by which of the following psychiatric terms

o. Delusion

p. Hallucination

q. Illusion

r. Projection

s. Synesthesia

308. All of the following are associated with a good prognosis in schizophrenia except

o. Sudden onset of symptoms

p. No affective symptoms

q. Confusion during psychosis

r. Severe precipitating stressor

s. Few premorbid schizoid traits

309. A 27-year-old woman was involved in a train derailment 2 weeks ago. Since that event,
she has felt down, has not slept well, has experienced repeated and intrusive thoughts of the
accident, and has recurrent nightmares. Lately, she has changed her commute to avoid the train,
even though this adds 3 hours to her commute daily. When on the train she has an acute
increase in her anxiety. She also often becomes “jumpy” whenever she hears the train going by
her home. Which of the following diagnoses is the most appropriate for this patient?

p. Acute stress disorder

q. Adjustment disorder

r. Generalized anxiety disorder (GAD)

s. Posttraumatic stress disorder (PTSD)

t. Major depressive disorder (MDD)

310. Features of anhedonia may include all of the following except

p. Difficulty describing or being aware of emotions

q. Derealization

r. Loss of pleasure

s. Inability to experience normal emotions

t. Withdrawal from interest

311. Psychomotor retardation is characterized by all of the following except

n. Paucity of spontaneous movements

o. Reduced speech amplitude and flow

p. Indecisiveness

q. Poor concentration

r. Restlessness

312. Double depression is characterized by

m. Two episodes of major depressive disorder per month consistently

n. Superimposed bipolar II disorder and atypical depression


o. Recurrent major depressive disorder superimposed with dysthymic disorder

p. Two family members suffering from major depressive disorder concurrently

q. Recurrent major depressive disorder with current symptoms twice as disabling as usual

313. Which of the following is the best predictor of likelihood of attempting suicide in future?

p. Gender

q. Alcohol abuse

r. Unemployment

s. Prior suicide attempt

t. Recent divorce

314. The risk of developing anxiety disorders is enhanced by

n. Eating disorders

o. Depression

p. Substance abuse

q. Allergies

r. All of the above

315. Which of the following examples or situations is most likely to cause PTSD?

p. Involvement in an earthquake

q. Being diagnosed with cancer

r. Rape

s. Witnessing a crime

t. Observing a flood
316. Unexpected panic attacks are required for the diagnosis of

p. panic disorder

q. social phobia

r. specific phobia

s. generalized anxiety disorder

t. all of the above

317. A 43-year-old man tells his psychiatrist that he is spending several hours in the morning
checking all the light switches to make sure they are off. He states that if he does not do this, he
is overcome with anxiety. This is an example of which of the following symptoms?

o. Catalepsy

p. Compulsions

q. Magical thinking

r. Anhedonia

318. A 24-year-old woman comes to the emergency room with the chief complaint that “my
stomach is rotting out from the inside.” She states that for the last six months she has been
crying on a daily basis and that she has decreased concentration, energy, and interest in her
usual hobbies. She has lost 25 lb during that time. She cannot get to sleep, and when she does,
she wakes up early in the morning. For the past three weeks, she has become convinced that she
is dying of cancer and is rotting on the inside of her body. Also, in the past two weeks she has
been hearing a voice calling her name when no one is around. Which of the following is the most
likely diagnosis?

n. Delusional disorder

o. Schizoaffective disorder

p. Schizophreniform disorder

q. Schizophrenia

r. Major depression with psychotic features


319. A 19-year-old man is brought to the physician by his parents after he called them from
college, terrified that the Mafia was after him. He reports that he has eaten nothing for the past
six weeks other than canned beans because “they are into everything—I can’t be too careful.”
He is convinced that the Mafia has put cameras in his dormitory room and that they are
watching his every move. He occasionally hears the voices of two men talking about him when
no one is around. His roommate states that for the past two months the patient has been
increasingly withdrawn and suspicious. Which of the following is the most likely diagnosis?

q. Delusional disorder

r. Schizoaffective disorder

s. Schizophreniform disorder

t. Schizophrenia

u. PCP intoxication

320. A 36-year-old woman is brought to the psychiatrist by her husband because for the past
eight months she has refused to go out of the house, believing that the neighbors are trying to
harm her. She is afraid that if they see her they will hurt her, and she finds many small bits of
evidence to support this. This evidence includes the neighbors’ leaving their garbage cans out on
the street to try to trip her, parking their cars in their driveways so they can hide behind them
and spy on her, and walking by her house to try to get a look into where she is hiding. She states
that her mood is fine and would be “better if they would leave me alone.” She denies hearing
the neighbors or anyone else talk to her, but is sure that they are out to “cause her death and
mayhem.” Which of the following is the most likely diagnosis?

o. Delusional disorder

p. Schizophreniform disorder

q. Schizoaffective disorder

r. Schizophrenia

s. Major depression with psychotic features

321. A 55-year-old man is brought to the psychiatrist by his wife after she found him
wandering outside their home wearing only his underwear. On exam, the patient notes that his
memory is “not as good as it used to be.” Which of the following tests is most likely to be helpful
in the diagnosis of this patient?
o. EEG

p. MRI

q. Serum glucose

r. Serum amylase

s. Urinary myoglobin

322. A 65-year-old man, who had been hospitalized for an acute pneumonia three days
previously, begins screaming for his nurse, stating that “there are people in the room out to get
me.” He then gets out of bed and begins pulling out his IV line. On exam, he alternates between
agitation and somnolence. He is not oriented to time or place. His vital signs are as follows:
pulse, 126 beats per minute; respiration, 32 per minute; blood pressure (BP), 80/58;
temperature, 39.2C (102.5F). Which of the following diagnoses best fits this patient’s clinical
picture?

o. Dementia

p. Schizophreniform disorder

q. Fugue state

r. Delirium

s. Brief psychotic episode

323. A 29-year-old man is brought to the emergency room by his wife after he woke up with
paralysis of his right arm. The patient reports that the day before, he had gotten into a verbal
altercation with his mother over her intrusiveness in his life. The patient notes that he has
always had mixed feelings about his mother, but that people should always respect their
mothers above all else. Which of the following diagnoses best fits this patient’s clinical picture?

m. Major depression

n. Conversion disorder

o. Histrionic personality disorder

p. Fugue state

q. Adjustment disorder
324. A 28-year-old business executive sees her physician because she is having difficulty in
her new position, as it requires her to do frequent public speaking. She states that she is terrified
she will do or say something that will cause her extreme embarrassment. The patient says that
when she must speak in public, she becomes extremely anxious and her heart beats
uncontrollably. Based on this clinical picture, which of the following is the most likely diagnosis?

o. Panic disorder

p. Avoidant personality disorder

q. Specific phobia

r. Agoraphobia

s. Social phobia

325. A 24-year-old man comes to the physician with the chief complaint that his nose is too
big, to the point of being hideous. The patient states that his nose is a constant embarrassment
to him, and he would like it surgically reduced. He tells the physician that three previous
surgeons had refused to operate on him because they said his nose was fine, but the patient
asserts that “they just didn’t want such a difficult case.” The physician observes that the patient’s
nose is of normal size and shape. Based on this patient’s clinical picture, which of the following is
the most likely diagnosis?

n. Schizophrenia

o. Narcissistic personality disorder

p. Body dysmorphic disorder

q. Anxiety disorder not otherwise specified

r. Schizoaffective disorder

326. Which of the following statements about delusions is true?

k. They are found almost exclusively in schizophrenia


l. Grandiose delusions are rarely encountered except in mania

m. They involve a disturbance of thought content

n. They involve a disturbance in perception

o. They are a type of hallucination

327. An anxiety disorder is:

o. An emotional state identified by panic attacks.

p. An emotional condition classified by excessive checking.

q. Disordered thinking.

r. An excessive or aroused state characterized by feelings of apprehension, uncertainty and fear.

328. A binge-eating episode is associated with which of the following according to the DSM 5
diagnostic criteria for binge eating disorder?

n. Eating much more rapidly than normal

o. Eating until feeling uncomfortably ill

p. Eating large amounts of food when not feeling physically hungry

q. All of the above

329. A young woman, who has a very limited memory of her childhood years but knows that
she was removed from her parents because of their abuse and neglect, frequently cannot
account for hours or even days of her life. She hears voices that alternately plead, reprimand, or
simply comment on what she is doing. Occasionally, she does not remember how and when she
arrived at a specific location. She finds clothes she does not like in her closet, and she does not
remember having bought them. Her friends are puzzled because sometimes she acts in a
childish, dependent way and at other times becomes uncharacteristically aggressive and
controlling. These symptoms are commonly seen in which of the following disorders?

n. Dissociative amnesia

o. Depersonalization disorder
p. Korsakoff’s dementia

q. Dissociative identity disorder

r. Schizophrenia

330. A 54-year-old man with a chronic mental illness seems to be constantly chewing. He
does not wear dentures. His tongue darts in and out of his mouth, and he occasionally smacks
his lips. He also grimaces, frowns, and blinks excessively. Which of the following disorders is
most likely in this patient?

n. Tourette’s syndrome

o. Akathisia

p. Tardive dyskinesia

q. Parkinson’s disease

r. Huntington’s disease

331. During the hectic weeks preceding her wedding, a 22-year-old woman in good health
and without a previous history of psychiatric illness occasionally feels unreal and detached from
her own body, “like in a dream.” The episodes last a few minutes and resolve spontaneously.
Which of the following statements is correct?

n. These symptoms can occur in people without psychiatric disorders

o. The patient suffers from depersonalization disorder

p. Depersonalization symptoms are rare in other psychiatric disorders

q. The patient suffers from somatization disorder

r. The patient is malingering

332. For the past three years, a 24-year-old college student has suffered from chronic
headaches, fatigue, shortness of breath, dizziness, ringing ears, and constipation. He is incensed
when his primary physician recommends a psychiatric evaluation because no organic cause for
his symptoms could be found. Which of the following disorders is most likely in this patient?

p. Somatic symptom disorder

q. Specific phobia

r. Dissociative identity disorder

s. Obsessive-compulsive disorder

t. Dissociative fugue

u. Body dysmorphic disorder

v. Dysthymia

333. You are treating a 48-year-old married female on the inpatient medical unit for
pyelonephritis; she has responded well to appropriate antibiotic therapy and has been afebrile
for the last 24 hours. You inform her of likely discharge if she continues to improve. The next
morning, however, she complains of feeling feverish and achy, and having dysuria again. The
nursing staff reports that she has a sudden fever of 103°F. You treat the fever with
acetaminophen and perform a physical examination, order chest x-rays, draw blood, and order a
urinalysis with culture. While you are awaiting these results, the nurse informs you that she
witnessed the patient dipping her thermometer into a hot cup of tea before her temperature
was taken. Which of the following is the most likely diagnosis?

o. Factitious disorder

p. Functional neurological symptom disorder

q. Illness anxiety disorder

r. Malingering

s. Somatic symptom disorder

334. A 27-year-old woman comes to a psychiatrist with the chief complaint of feeling
depressed her entire life. While she states that she has never been so depressed that she was
unable to function, she never feels really good for more than a week or two at a time. She has
never been suicidal or psychotic, though her self-esteem is chronically low. Which of the
following is the most likely diagnosis?

l. Major depression

m. Adjustment disorder

n. Dysthymia

o. Bipolar disorder

335. A patient reports to you that for the past week or two he has had the belief that his
intestines and his heart have been removed. When asked about his lack of getting out in the
world, he responds “What world? There is no world!” This aspect of the patient’s illness would
best be referred to as which of the following?

o. schizoaffective disorder

p. Capgras syndrome

q. folie à deux

r. Cotard syndrome

s. major depression

Each of the questions or incomplete statements below is followed by several suggested responses or
completions. Select the one that is best in each case.

336. A patient with severe depression developed a delusion of being wicked and deserving
punishment. Which of the following answer is correct?

t. Delusion is primary symptom

u. Depression is secondary symptom

v. Delusion is secondary symptom

w. Both are primary symptoms

x. Both are secondary symptoms

337. Which of the following is not included in the disorders of the stream of thought?
s. Pressure of thought

t. Poverty of thought

u. thought perseveration

v. Made thoughts

w. Thought blocking

338. A 23-year-old woman comes to the emergency room with the chief complaint that she
has been hearing voices for seven months. Besides the hallucinations, she has the idea that the
radio is giving her special messages. When asked the meaning of the proverb “People in glass
houses should not throw stones,” the patient replies, “Because the windows would break.”
Which of the following mental status findings does this patient display?

r. Poverty of content

s. Concrete thinking

t. Flight of ideas

u. Loose associations

v. Autistic thinking

339. A 42-year-old man comes to the emergency room with the chief complaint that “the
men are following me.” He also complains of hearing a voice telling him to hurt others. He tells
the examiner that the news anchorman gives him special messages about the state of the world
every night through the TV. This last belief is an example of which of the following psychiatric
findings?

t. Grandiose delusion

u. Illusion

v. Loose association

w. Idea of reference

x. Clouding of consciousness
340. Stereotypy is

u. temporary loss of muscle tone and weakness precipitated by a variety of emotional states

v. pathological imitation of movements of one person by another

w. ingrained, habitual involuntary movement

x. repetitive fixed pattern of physical action or speech

y. subjective feeling

341. A 17-year-old boy is diagnosed with schizophrenia. What is the risk that one of his
siblings will develop the disease?

s. 2%

t. 5%

u. 10%

v. 20%

w. 30%

342. A 56-year-old man is brought to the physician’s office by his wife because she has noted
a personality change during the past three months. While the patient is being interviewed, he
answers every question with the same three words. Which of the following symptoms best fits
this patient’s behavior

t. Negative symptoms

u. Disorientation

v. Concrete thinking

w. Perseveration
343. A 19-year-old man is brought to the physician by his parents after he called them from
college, terrified that the Mafia was after him. He reports that he has eaten nothing for the past
six weeks other than canned beans because “they are into everything—I can’t be too careful.”
He is convinced that the Mafia has put cameras in his dormitory room and that they are
watching his every move. He occasionally hears the voices of two men talking about him when
no one is around. His roommate states that for the past two months the patient has been
increasingly withdrawn and suspicious. Which of the following is the most likely diagnosis?

s. Delusional disorder

t. Schizoaffective disorder

u. Schizophreniform disorder

v. Schizophrenia

w. PCP intoxication

344. A 69-year-old man is brought to see his physician by his wife. She notes that over the
past year he has experienced a slow, stepwise decline in his cognitive functioning. One year ago
she felt his thinking was “as good as it always had been,” but now he gets lost around the house
and can’t remember simple directions. The patient insists that he feels fine, though he is
depressed about his loss of memory. He is eating and sleeping well. Which of the following is the
most likely diagnosis?

u. Multi-infarct dementia

v. Mood disorder secondary to a general medical condition

w. Schizoaffective disorder

x. Delirium

y. Major depression

345. A psychiatric resident is called to consult on the case of a 75-year-old woman who had
undergone a hip replacement two days before. On examination, the resident notes that the
patient states the date as 1956, and she thinks she is at her son’s house. These impairments
illustrate which aspect of the mental status examination?

s. Concentration
t. Memory

u. Thought process

v. Orientation

w. Level of consciousness

346. The negative symptoms of schizophrenia include all of the following except

t. alogia

u. affective flattening

v. avolition

w. aggressivity

x. inattentiveness

347. Erotomania, the delusional disorder in which the person makes repeated efforts to
contact the object of the delusion, through letter, phone call, and stalking, is also referred to as

t. Cotard’s syndrome

u. Clérambault’s syndrome

v. Fregoli’s syndrome

w. Ganser’s syndrome

x. Capgras’s syndrome

348. A 20-year-old woman is brought to the emergency room by her family because they
have been unable to get her to eat or drink anything for the past two days. The patient, although
awake, is completely unresponsive both vocally and nonverbally. She actively resists any attempt
to be moved. Her family reports that during the previous seven months she became increasingly
withdrawn, socially isolated, and bizarre, often speaking to people no one else could see. Which
of the following is the most likely diagnosis?

w. Schizoaffective disorder

x. Delusional disorder

y. Schizophreniform disorder

z. Catatonia

349. A 52-year-old man is sent to see a psychiatrist after he is disciplined at his job because
he consistently turns in his assignments late. He insists that he is not about to turn in anything
until it is “perfect, unlike all of my colleagues.” He has few friends because he annoys them with
his demands for “precise timeliness” and because of his lack of emotional warmth. This has been
a lifelong pattern for the patient, though he refuses to believe the problems have anything to do
with his personal behavior. Which of the following is the most likely diagnosis for this patient?

u. Obsessive-compulsive disorder

v. Obsessive-compulsive personality disorder

w. Borderline personality disorder

x. Bipolar disorder, mixed state

y. Anxiety disorder not otherwise specified

350. A 27-year-old woman comes to a psychiatrist with the chief complaint of feeling
depressed her entire life. While she states that she has never been so depressed that she was
unable to function, she never feels really good for more than a week or two at a time. She has
never been suicidal or psychotic, though her self-esteem is chronically low. Which of the
following is the most likely diagnosis?

t. Major depression

u. Adjustment disorder

v. Dysthymia
w. Bipolar disorder

351. A 56-year-old man has been hospitalized for a myocardial infarction. Two days after
admission, he awakens in the middle of the night and screams that there is a man standing by
the window in his room. When the nurse enters the room and turns on a light, the patient is
relieved to learn that the “man” was actually a drape by the window. This misperception of
reality is best described by which of the following psychiatric terms

t. Delusion

u. Hallucination

v. Illusion

w. Projection

x. Synesthesia

352. All of the following are associated with a good prognosis in schizophrenia except

s. Sudden onset of symptoms

t. No affective symptoms

u. Confusion during psychosis

v. Severe precipitating stressor

w. Few premorbid schizoid traits

353. A patient reports to you that for the past week or two he has had the belief that his
intestines and his heart have been removed. When asked about his lack of getting out in the
world, he responds “What world? There is no world!” This aspect of the patient’s illness would
best be referred to as which of the following?
u. schizoaffective disorder

v. Capgras syndrome

w. folie à deux

x. Cotard syndrome

y. major depression

354. A 27-year-old woman was involved in a train derailment 2 weeks ago. Since that event,
she has felt down, has not slept well, has experienced repeated and intrusive thoughts of the
accident, and has recurrent nightmares. Lately, she has changed her commute to avoid the train,
even though this adds 3 hours to her commute daily. When on the train she has an acute
increase in her anxiety. She also often becomes “jumpy” whenever she hears the train going by
her home. Which of the following diagnoses is the most appropriate for this patient?

t. Acute stress disorder

u. Adjustment disorder

v. Generalized anxiety disorder (GAD)

w. Posttraumatic stress disorder (PTSD)

x. Major depressive disorder (MDD)

355. A49-year-old bank teller without a psychiatric history is referred to your office for the
first time by her internist for an evaluation. For the past 2 months, she has been increasingly
convinced that a well-known pop music star is in love with her and that they have had an
ongoing affair. She is well-groomed, and there is no evidence of thought disorder or
hallucinations. Her husband reveals that she has been functioning well at work and in other
social relationships. Which of the following is the most likely diagnosis?

t. delusional disorder

u. acute reactive psychosis

v. prodromal schizophrenia

w. paranoid personality disorder


x. schizophreniform disorder

356. Features of anhedonia may include all of the following except

t. Difficulty describing or being aware of emotions

u. Derealization

v. Loss of pleasure

w. Inability to experience normal emotions

x. Withdrawal from interest

357. Psychomotor retardation is characterized by all of the following except

t. Paucity of spontaneous movements

u. Reduced speech amplitude and flow

v. Indecisiveness

w. Poor concentration

x. Restlessness

358. Double depression is characterized by

t. Two episodes of major depressive disorder per month consistently

u. Superimposed bipolar II disorder and atypical depression

v. Recurrent major depressive disorder superimposed with dysthymic disorder

w. Two family members suffering from major depressive disorder concurrently

x. Recurrent major depressive disorder with current symptoms twice as disabling as usual

359. Which of the following is the best predictor of likelihood of attempting suicide in future?
u. Gender

v. Alcohol abuse

w. Unemployment

x. Prior suicide attempt

y. Recent divorce

360. The risk of developing anxiety disorders is enhanced by

s. Eating disorders

t. Depression

u. Substance abuse

v. Allergies

y. All of the above

361. Which of the following examples or situations is most likely to cause PTSD?

s. Involvement in an earthquake

t. Being diagnosed with cancer

u. Rape

v. Witnessing a crime

w. Observing a flood

362. Unexpected panic attacks are required for the diagnosis of

u. panic disorder

v. social phobia

w. specific phobia

x. generalized anxiety disorder


y. all of the above

363. A 43-year-old man tells his psychiatrist that he is spending several hours in the morning
checking all the light switches to make sure they are off. He states that if he does not do this, he
is overcome with anxiety. This is an example of which of the following symptoms?

p. Catalepsy

q. Compulsions

r. Magical thinking

s. Anhedonia

364. A 24-year-old woman comes to the emergency room with the chief complaint that “my
stomach is rotting out from the inside.” She states that for the last six months she has been
crying on a daily basis and that she has decreased concentration, energy, and interest in her
usual hobbies. She has lost 25 lb during that time. She cannot get to sleep, and when she does,
she wakes up early in the morning. For the past three weeks, she has become convinced that she
is dying of cancer and is rotting on the inside of her body. Also, in the past two weeks she has
been hearing a voice calling her name when no one is around. Which of the following is the most
likely diagnosis?

r. Delusional disorder

s. Schizoaffective disorder

t. Schizophreniform disorder

u. Schizophrenia

v. Major depression with psychotic features

365. A 32-year-old woman is brought to the emergency room by the police after she was
found standing in the middle of a busy highway, naked, commanding the traffic to stop. In the
emergency room she is agitated and restless, with pressured speech and an affect that
alternates between euphoric and irritable. Her father is contacted and states that this kind of
behavior runs in the family. Which of the following is the most likely diagnosis?

s. Delirium
t. Bipolar disorder, manic

u. Bipolar disorder, mixed state

v. Cyclothymia

w. Schizophrenia

366. A 52-year-old man comes to the physician with the chief complaint of feeling depressed
for the past two months. He notes that he is not sleeping well, has lost 25 lb in the last six
weeks, and is experiencing anergia and anhedonia. In addition, in the past four weeks he has
begun to hear the voice of his dead father telling him that he is a failure and has begun worrying
that his organs are rotting away. Which of the following statements is true?

p. The patient should be started on an SSRI and an antipsychotic

q. The patient is having an acute schizophrenic episode

r. The patient is likely suffering from a factitious disorder with psychological

s. symptoms

t. The patient is likely abusing alcohol

u. The patient should be started on an SSRI alone

367. A 55-year-old man is brought to the psychiatrist by his wife after she found him
wandering outside their home wearing only his underwear. On exam, the patient notes that his
memory is “not as good as it used to be.” Which of the following tests is most likely to be helpful
in the diagnosis of this patient?

u. EEG

v. MRI

w. Serum glucose

x. Serum amylase

y. Urinary myoglobin
368. A 65-year-old man, who had been hospitalized for an acute pneumonia three days
previously, begins screaming for his nurse, stating that “there are people in the room out to get
me.” He then gets out of bed and begins pulling out his IV line. On exam, he alternates between
agitation and somnolence. He is not oriented to time or place. His vital signs are as follows:
pulse, 126 beats per minute; respiration, 32 per minute; blood pressure (BP), 80/58;
temperature, 39.2C (102.5F). Which of the following diagnoses best fits this patient’s clinical
picture?

u. Dementia

v. Schizophreniform disorder

w. Fugue state

x. Delirium

y. Brief psychotic episode

369. A 29-year-old man is brought to the emergency room by his wife after he woke up with
paralysis of his right arm. The patient reports that the day before, he had gotten into a verbal
altercation with his mother over her intrusiveness in his life. The patient notes that he has
always had mixed feelings about his mother, but that people should always respect their
mothers above all else. Which of the following diagnoses best fits this patient’s clinical picture?

s. Major depression

t. Conversion disorder

u. Histrionic personality disorder

v. Fugue state

w. Adjustment disorder

370. A 34-year-old secretary climbs 12 flights of stairs every day to reach her office because
she is terrified by the thought of being trapped in the elevator. She has never had any traumatic
event occur in an elevator; nonetheless, she has been terrified of them since childhood. Which
of the following is the most likely diagnosis?

m. Social phobia

n. Performance anxiety
o. Generalized anxiety disorder

p. Specific phobia

q. Agoraphobia

371. Which of the following statements about delusions is true?

v. They are found almost exclusively in schizophrenia

w. Grandiose delusions are rarely encountered except in mania

x. They involve a disturbance of thought content

y. They involve a disturbance in perception

z. They are a type of hallucination

372. An anxiety disorder is:

t. An emotional state identified by panic attacks

u. An emotional condition classified by excessive checking

v. Disordered thinking

w. An excessive or aroused state characterized by feelings of apprehension, uncertainty and fear

373. A binge-eating episode is associated with which of the following according to the DSM 5
diagnostic criteria for binge eating disorder?

t. Eating much more rapidly than normal

u. Eating until feeling uncomfortably ill

v. Eating large amounts of food when not feeling physically hungry

w. All of the above


374. A 54-year-old man with a chronic mental illness seems to be constantly chewing. He
does not wear dentures. His tongue darts in and out of his mouth, and he occasionally smacks
his lips. He also grimaces, frowns, and blinks excessively. Which of the following disorders is
most likely in this patient?

t. Tourette’s syndrome

u. Akathisia

v. Tardive dyskinesia

w. Parkinson’s disease

x. Huntington’s disease

375. For the past three years, a 24-year-old college student has suffered from chronic
headaches, fatigue, shortness of breath, dizziness, ringing ears, and constipation. He is incensed
when his primary physician recommends a psychiatric evaluation because no organic cause for
his symptoms could be found. Which of the following disorders is most likely in this patient?

r. Somatic symptom disorder

s. Specific phobia

t. Obsessive-compulsive disorder

u. Dissociative fugue

v. Body dysmorphic disorder

w. Dysthymia

Patient believes that someone close to him or her has been replaced by an exact double
x. ans- delusion of doubles
A loss of the normal structure of thinking. - Loosening of associations

• Never any history of acute mania


1. Bipolar II
2. B. Bipolar I
3. C. Both
4. D. Cyclothymic disorder
• At least 2 years periods with hypomanic symptoms
• Bipolar II
• B. Bipolar I
• C. Both
D. Cyclothymic disorder
• Symptoms of hypomania are present
• Bipolar II
• B. Bipolar I
• C. Both
D. Cyclothymic disorder
• At least one manic episode is present
• Bipolar II
• B. Bipolar I
• C. Both
D. Cyclothymic disorder

2. She was started on appropriate treatment. One year • Schizophrenia


later this woman returns to your office with her • Delusional disorder
mother for follow-up. Her symptoms remitted within a • Bipolar disorder, manic phase
month. However, she has not done well in her • Schizoaffective disorder
freshman year and for the past several months has • Schizophreniform disorder
continued to experience worsening social isolation •
and amotivation. While she has not used any
substances since she last saw you, she reluctantly
admits to occasionally hearing the devil
communicating with her. She tries to ignore the
communication, and has taken to arranging her books
in a certain manner to prevent his controlling her
thoughts. On her mental status examination she
makes poor eye contact and her affect is blunted. Her
mother reports that the patient now rarely calls home,
though before she’d do so twice weekly. Which of the
following is the most likely diagnosis?

You might also like